Ortho Review Questions

¡Supera tus tareas y exámenes ahora con Quizwiz!

lower back pain population

Back pain is most common between the ages of 30 and 55 years with prevalence rates similar for men and women, although 50 to 90% of pregnant women develop low back pain.

hat secondary malignancy is commonly associated with radiation therapy?

Bone and soft tissue sarcoma

De Quervain tenosynovitis is a common condition that occurs in patients who have experienced excessive use of the thumb or wrist. Finklestein Tests what

Extensor pollicis brevis and abductor pollicis longus

Which grading system is followed for osteoarthritis?

Kellgren-Lawrence grading system

Morton neuroma

Morton neuroma is a perineural fibrosis of the common digital nerve as it passes between the metatarsal heads. The fibrosis is secondary to repetitive irritation of the nerve. The condition is most common between the third and fourth toes (web space). It has a female to male ratio of 5:1

Pseudomonas aeruginosa causes more than 90% of foot osteochondritis in plantar puncture wounds. What bacteria would most likely cause this complication?

Pseudomonas aeruginosa causes more than 90% of foot osteochondritis in plantar puncture wounds.

glenohumeral joint instability

Sulcus sign

Radiculopathy due to nerve root compression occurs most commonly at which nerve root within the brachial plexus?

The C7 nerve root is affected the most often (approximately 45-60%).

Frozen Shoulder

This is adhesive capsulitis (frozen shoulder). MRI arthrogram can substantiate a frozen shoulder by demonstrating a contracted capsule and loss of the inferior pouch of the capsule

Sinding-Larsen-Johansson

is not correct because the patient describes pain around the tibial tubercle and not the patellar tendon, which is the structure associated with Sinding-Larsen-Johansson.

In a 4-year-old female child who presents with "toeing in," which of the following is the likely etiology?

"Toeing in" in children before the age of 2 is typically due to tibial torsion; however, any "toeing in" after the age of 2 to 3, is usually due to femoral anteversion.

You are getting ready to evaluate a patient who has a past history of gout. The chief complaint is that the patients gout has flared up again, causing pain. Based on your knowledge of gout, which joint is most commonly the site of an initial gout attack?

1st metatarsaphalangeal joint

what test would best confirm the diagnosis of cerebral systemic lupus erythematosus?

Antinuclear antibody (ANA) testing is used for serologic testing for SLE. There are 2 major types of ANA: autoantibodies to DNA and histones and autoantibodies to extractable nuclear antigens. anti-dsDNA antibodies autoantibody to Smith antigen

What type of fracture is not related to an acute bony trauma? A Greenstick B Stress C Oblique D Comminuted E Spiral

A stress or fatigue fracture is caused by small, repetitive forces that usually involve the metatarsal shafts, the distal tibia, and the femoral neck (though many other bones may be affected). These fractures may not be seen on initial radiographs. A greenstick fracture is an incomplete traumatic fracture with angular deformity seen in children. An oblique fracture is a traumatic fracture with an angulated fracture line. A comminuted fracture is a traumatic fracture in which there are more than two fracture segments. A spiral fracture is a traumatic fracture that has a multiplanar and complex fracture line usually caused by an excessive rotational force on a bone.

Eversion ankle sprains

are also termed high ankle sprains. This injury is commonly acquired after the foot being turned out or externally rotated and everted, as during a tackle. Pain associated with this sprain is typically worse than with other sprains. Mild swelling or even an effusion may be present. No dislocation is apparent.

A 34-year-old woman presents to your clinic with a 3-month history of a mildly tender mass on the dorsocentral aspect of her dominant right wrist. She says the "lump comes and goes," but this time, it has stayed and become tender. She denies any known trauma of the wrist or hand. What examination technique would help to show that this mass is a ganglion cyst?

A prominent ganglion cyst of the dorsal wrist will transilluminate while a solid tumor will not

Which of the following patterns of stiffness is most characteristic of patients with rheumatoid arthritis?

morning stiffness greater than 1 hour

Which of the following clinical manifestations is most characteristic of polymyalgia rheumatica (PMR)?

An abrupt onset of proximal muscle pain and stiffness in the shoulder and pelvic girdle areas, usually associated with fever, malaise, and weight loss, is characteristic of polymyalgia rheumatica. Subcutaneous inflammatory lesions denote erythema nodosum. Symmetrical weakness initially in the legs that progresses caudally is characteristic of Guillain-Barré syndrome.

Children and aspirin

Aspirin-taking in children has been linked to Reyes syndrome, a rare syndrome in which encephalopathy occurs and if not diagnosed early can lead to death. Advice used to state that no child under 12 should take aspirin. However, due to the wide availability of aspirin, and the availability of other safer treatment choices it is now recommended that children under 18 should not take aspirin. Many parents might not be aware that the age restriction for aspirin has changed and it is important that pharmacists remind parents

osteosarcoma

most common malignant bone tumor in the pediatric population, and it is thought to arise from primitive mesenchymal, bone-forming cells The 3 most common sites of origin for osteosarcoma are the femur (40%), tibia (20%), and proximal humerus (10%). Metastasis to the lungs is the most common;

Patients in which of the following age groups are least likely to experience a dislocation or sprain when a significant stress is placed on their joints?

Dislocations and ligamentous injuries are uncommon in prepubertal children as the ligaments and joints are quite strong as compared to the adjoining growth plates. Excessive force applied to a child's joint is more likely to cause a fracture through the growth plate than a dislocation or sprain

osteoarthritis

Knee radiography is typically expected to reveal joint space narrowing (especially of the medial compartment), thickened, dense subchondral sclerosis, osteophytes, and bony spurring.

Marfan's syndrome

Scoliosis and kyphoscoliosis are seen with Marfan's syndrome. Kyphoscoliosis consists of both kyphosis and scoliosis. Scoliosis is an abnormal lateral curvature of the spine. Kyphosis is a spinal deformity due to extensive flexion. It is an exaggeration of the thoracic curvature. This produces a "hunchback."

branous glomerulonephritis is associated with which of the following conditions?

Systemic lupus erythematosus

foot in diabetic patients

The patient has osteomyelitis. Staphylococcus auerus is the most common cause of osteomyelitis of the foot in diabetic patients.

Inflammation or arthritis of the acromioclavicular (AC) joint

This condition usually presents with localized tenderness over the AC joint or pain with adduction of the affected shoulder. The "crossover test" is also used to assess the AC joint. Therefore, a "positive crossover test" is when adduction of the patient's affected arm across the chest elicits pain. The patient above had a "negative crossover test".

A 20-year-old man presents complaining of right-hand pain after punching a wall in anger the day before. He complains of pain on the ulnar aspect side of his hand. On physical exam, he has marked swelling and ecchymosis of the ulnar side of his hand. The radiograph is below. Which of the following is not true regarding this injury? The patient probably has a boxer's fracture, or fracture of the neck of the fifth metacarpal. The greater mobility of the fourth and fifth metacarpals makes them more likely to be fractured than the other metacarpals. Emergent management includes immobilization in an ulnar gutter splint in the position of function. Avascular necrosis usually occurs with this injury.

D. Avascular necrosis is not common with this injury, but it is common with navicular or scaphoid fractures and lunate fractures in the carpus. The concern with these fractures is the possibility of rotational deformities. The fifth metacarpal is the most commonly fractured metacarpal.

A 10-year-old boy was tackled by his brother while playing football. He immediately felt pain near his left shoulder. He is having trouble moving his shoulder as well. On physical exam, he has palpable tenderness medial to his shoulder about halfway over toward his neck. His shoulder is not swollen, and he can use his elbow without difficulty. However, he does have what looks like a swollen lump halfway between his shoulder and his neck anteriorally. His radiograph is shown below. Which of the following is the most likely diagnosis? Rotator-cuff tear Posterior shoulder dislocation Humeral fracture Clavicle fracture

D. The clavicle is the most commonly fractured bone of childhood. A direct blow to the shoulder often fractures the clavicle. Posterior shoulder dislocations are rare, but are commonly seen in patients who have suffered seizures or electric shock. The rotator-cuff muscles are the supraspinatus, infraspinatus, subscapularis, and teres minor. Rotator-cuff injuries generally result from degenerative changes with age and occur less likely secondary to trauma or a sports injury. Humeral fractures have swelling down the arm, not near the clavicle. This patient also has suffered an acromioclavicular (AC) separation due to his injuries.

A 20-year-old wrestler presents to the emergency department after injuring his shoulder. He fell backward during practice and put his right hand behind him to break his fall. He has injured this same shoulder several times in the past. He is in severe pain. He cannot move the shoulder and is holding his right arm slightly abducted. He is reluctant to let you passively move the shoulder and seems to have numbness over the lateral aspect of the right shoulder region as well. The radiograph is shown below. Which of the following is the most likely diagnosis? Fracture of the distal clavicle Calcific tendonitis Biceps tendon rupture Anterior shoulder dislocation

D. The glenohumeral joint is the most commonly dislocated major joint in the body. Anterior dislocations account for over 95% of all shoulder dislocations. The mechanism for this injury may be surprisingly minor, and the recurrence rate is high in young patients with this condition. A fractured clavicle is possible, but in such a case, the patient can usually move the shoulder. Calcific tendonitis is not a traumatic one-time injury. Biceps tendon rupture is usually a disease of mature adults, and complaints often are felt further down the arm.

The mother of a 3-year-old boy states that her child does not walk normally. The mother feels that the child has been falling more frequently; she also tells you that his symptoms are worsening quickly. On exam, the child uses his hands to push himself into an upright position from sitting on the ground. He walks with a waddling gait. There is hypertrophy of the bilateral calves, lumbar lordosis, and hyporeflexia. There is also cognitive delay. His labs reveal an increased creatine kinase. Question What is the most likely diagnosis?

Duchenne muscular dystrophy is the correct response; symptoms are usually noted between the ages of 3 to 5 years in boys. The disease is present at birth; however, symptoms often are not noted until the child starts walking. A waddling gait, hypertrophy of the calves, lumbar lordosis, and hyporeflexia are all common exam findings with this disease. The way in which the child pushed himself into an upright position is known as Gower's sign. There is also an increase in creatine kinase. This is the most common form of muscular dystrophy in children. Facioscapulohumeral muscular dystrophy is incorrect; this disease usually starts later in the teenage years. Weakness is usually noted in the face, arms, and around the shoulders. Facial weakness is usually the initial sign of this disease.

Which of the following would be the best type of exercise to do to strengthen bones and prevent osteoporosis? A Ballistic stretching B Recumbent bike C Static stretching D Swimming E Weight training

E

A 46-year-old female presents with pain to her left wrist. She complains that it is painful and swollen as she points to the volar aspect of the wrist on the radial side. On examination, there is a small, soft bump on the dorsum of her wrist with a jelly-like consistency. What is the most likely diagnosis? A Cancerous tumor B Fracture C Ganglion cyst D Hematoma E Lipoma

Ganglion cysts commonly occur on the dorsal or volar aspect of the wrist. They result when a joint capsule or tendon sheath is damaged, allowing synovial fluid to escape producing a one-way valve, which allows fluid into the cyst, but not back out. The accumulating fluid forms the ganglion cyst. These cysts may or may not be tender and can fluctuate in size depending on activity level of the affected extremity. Cancerous tumors would tend to be much more firm, but also may be relatively pain free. Fractures would generally be exquisitely tender and if the bump is due to a displaced bone, it would be much more firm than a ganglion cyst. Hematomas are generally associated with acute trauma and would be tender and ecchymotic in many situations. Lipomas are benign fatty tumors that are more commonly seen on the thenar eminence than the dorsum of the wrist and their size does not change based on activity level.

Giant cell (temporal) arteritis

Giant cell (temporal) arteritis (D) is characterized by headache, jaw claudication, polymyalgia rheumatica, visual abnormalities, and a markedly elevated ESR. Giant cell arteritis is a systemic panarteritis affecting medium-sized and large vessels in patients over the age of 50. Giant cell arteritis is also called temporal arteritis because that artery is frequently involved. About 50% of patients with giant cell arteritis also have polymyalgia rheumatica. The classic symptoms suggesting that a patient has arteritis are headache, scalp tenderness, visual symptoms (particularly amaurosis fugax or diplopia), jaw claudication, or throat pain. Of these symptoms, jaw claudication has the highest positive predictive value. Polymyalgia rheumatica is a clinical diagnosis based on pain and stiffness of the shoulder and pelvic girdle areas, frequently in association with fever, malaise, and weight loss

A 25-year-old sexually active woman presents with a 2-day history of pain and swelling of her dorsal right wrist and fingers. For the past week, this pain and swelling has been in different joints of her body, including her left knee, left elbow, and right ankle. On exam, you note edema, effusion, and erythema over the dorsal right wrist with the wrist held in 15 degrees of extension. It is very tender on palpation and has virtually no range of motion secondary to the pain. You aspirate synovial fluid for lab studies. The lab studies reveal the following information: Synovial fluid: WBCs - 57,000 cells/mcL; Gram stain: too numerous to count WBCs with no bacteria seen; Culture: pending

Gonococcal septic arthritis, which this woman most likely has based on age, history, and physical and lab findings, should be admitted for IV ceftriaxone (Rocephin) until culture results return and have sensitivities determined

A 23-year-old Caucasian man presents with lower back pain. The pain has been occurring for several months. The pain is worst in the morning and improves with activity. He has associated stiffness. There is no known injury to the area, and the pain has not responded to rest and heat. He has also noticed blurred vision and photophobia. On examination, there is decreased range of motion in the back. Uveitis is noted. Question Based on the patient's findings, which laboratory study would likely be positive?

HLA-B27 is correct. The patient has lower back pain with uveitis, which is suggestive of ankylosing spondylitis. HLA-B27 is positive in 90% of Caucasian patients and 50% of African-American patients with ankylosing spondylitis. Anti-CCP antibodies and rheumatoid factor are incorrect. Anti-CCP antibodies and rheumatoid factor are found in rheumatoid arthritis and will be negative in ankylosing spondylitits.

: A 64-year-old female who has a history of injectable drug use presents with blood work that reveals leukocytosis with a left shift, and there is suspicion of osteomyelitis based on the patient's prior history. Based on this history, what bone would be most affected by hematogenous osteomyelitis in adults?

Hematogenous osteomyelitis accounts for about 20% of all cases of osteomyelitis in adults. It is more common in males and the prevalence is higher amongst those who are IV drug abusers, patients being treated with dialysis or who have sickle cell disease. Other conditions which may lead to sepsis (i.e. patients with central lines, urinary infections, and urethral catheterization) increase the risk of hematogenous osteomyelitis. Unlike children, the long bones are rarely affected in adults with the vertebrae being the most likely location for the bone infection to occur. Lumbar vertebrae are most often affected, followed by thoracic and cervical vertebrae.

a motor vehicle accident. The ER doctor suspects that the left shoulder pain is secondary to splenic rupture. What is this sign called?

Kehr's sign is pain in the left shoulder secondary to splenic rupture. Cullen's sign is ecchymosis around the umbilicus that can sometimes be seen with acute pancreatitis. Grey Turner's sign is ecchymosis of the flanks that can sometimes be seen with acute pancreatitis. Lisker's sign is tibial bone tenderness that can sometimes be elicited with deep vein thrombosis. Blumberg's sign is abdominal rebound tenderness. This is an indication of peritoneal irritation

n 83-year-old malnourished woman presents for examination. She stands with her shoulders rounded and has an exaggerated thoracic convexity. Question What type of deformity of the spine does she have?

Kyphosis is the posterior (backward) curvature of the spine as viewed from the side. It is a rounded thoracic convexity. This is common in aging patients and in women. Scoliosis is the lateral curvature of the spinal column. This can be functional or structural when it compensates for other abnormalities. Scoliokyphosis is the combined lateral and posterior curvature of the spine. Lordosis is the curvature of the lumbar spine. It is the accentuation of the normal lumbar curve. It may be as a compensation for other deformities, e.g. kyphosis.

A 28-year-old male involved in a motor vehicle accident has a direct impact injury to the dashboard of his car, striking his knee. His radiograph is shown below. Which of the following is the most pressing concern for this patient? Save Image Enlarge Image Avascular necrosis of the femoral head Chronic greater trochanteric bursitis Eventual fatigue fracture with a return to sporting activities Fibromyositis

The correct answer is A. Avascular necrosis refers to diminished blood supply causing eventual death of areas of tissue or bone, in this case, the femoral head. This traumatic dislocation of the patient's hip might compromise the blood supply of the vessels of the ligamentum teres through the fovea centralis and the retinaculum of Weitbrecht leading to the femoral head, eventually causing bone death and collapse of the subchondral bone. Trochanteric bursitis is inflammation of the bursa (a padlike sac lined with a synovial membrane that contains fluid that reduces friction between a tendon and a bone) overlying the proximal femoral thick process projecting upward externally to the union of the neck and shaft of the femur (hip). A fatigue fracture is a fine hairline fracture that develops from repetitive microtrauma, such as distance running. Fibromyositis is chronic pain in muscles and soft tissues surrounding joints.

A 70-year-old female presents having slipped on the ice in front of her home, falling onto her outstretched palm. She complains of pain and decreased range of motion of her wrist. Her radiograph is shown below. Which of the following is the most likely diagnosis? Save Image Enlarge Image Colles' fracture Smith's fracture Scaphoid fracture Boxer's fracture

The correct answer is A. Colles' fracture is a displaced dorsally angulated (apex volar) extra-articular fracture of the distal radius (wrist). Smith's fracture is a displaced volarly angulated (apex dorsally) extra-articular fracture of the distal radius (wrist). Potential complications with this fracture presentation include median nerve damage and radial artery damage. A displaced fracture needs anatomical realignment. A scaphoid fracture typically presents with pain over the anatomical snuffbox and usually follows a forced abduction of the thumb. A boxer's fracture is aptly named due to the fact it usually follows an injury in which the patient strikes an object (such as a wall or a mandible) and sustains a fracture of the fifth metacarpal.

A left-hand dominant male mechanic presents complaining of left volar wrist pain and paresthesias, primarily in his left thumb, index and middle fingers, that awaken him at night. He notes diminished grip strength. Considerable weakness is noted when the patient attempts to abduct his left thumb against resistance (as shown below). Which of the following is the most likely diagnosis? Save Image Enlarge Image Carpal tunnel syndrome De Quervain's tenosynovitis Ganglion cyst Osteoarthritis

The correct answer is A. Tests for carpal tunnel syndrome include Phalen's test (wrists actively held in full flexion, see below); Tinel's test (percussion of the medial nerve along its course at the volar wrist, see below); and thumb abduction against resistance. The thumb abduction maneuver tests the strength of the abductor pollicis brevis muscle, innervated solely by the median nerve. Ganglion cysts typically appear on the dorsum of the wrist. They become more prominent with wrist flexion and are often asymptomatic. De Quervain's tenosynovitis produces a positive Finkelstein's test, which is pain elicited when a patient flexes the thumb in a closed fist and the hand is deviated in the ulnar direction.

A 6-year-old female child presents with complaints of chronic hip pain so severe that she has not been able to walk to the school bus. Examination shows severe tenderness at the left hip with markedly decreased active and passive range of motion. Radiologic examination demonstrates joint effusion with widening. Which of the following is the most likely diagnosis?

Legg-Calvé-Perthes disease is also known as avascular necrosis of the proximal femur. It typically occurs in children between 4 and 8 years old and persistent hip pain is the main symptom. On examination, the clinician notices a limp and/or limitation of motion of the affected hip. Radiologic examination demonstrates the necrosis with effusion and joint space widening with a negative aspirate. Treatment involves surgical hip replacement. Slipped capital femoral epiphysis (SCFE) is due to the displacement of the proximal femoral epiphysis owing to disruption of the growth plate. The head is normally displaced medially and posteriorly relative to the femoral neck. It typically occurs in adolescence, specifically obese males, and can also be associated with hypothyroidism. SCFE usually occurs after direct trauma to the hip or a fall. Patients complain of vague symptoms at first that progress into pain of the hip or of the knee. On examination, there is decreased internal rotation of the hip that can be confirmed by lateral X-ray of the hip

: A 36-year-old auto mechanic presents to the emergency department after hurting his back on the job. While lifting an object, he experienced sudden pain in his lower back with radiation to the right buttock. He was initially treated for muscle strain with a nonsteroidal anti-inflammatory drug (NSAID) after x-rays of his lumbosacral spine demonstrated no pathology. He continued to complain of this low back pain now radiating posteriorly down his left leg to the mid-thigh. Physical examination is unremarkable. The most likely diagnosis is A lumbosacral strain B left S1 radiculopathy C cauda equina syndrome D L5-S1 disc herniation E lateral femoral cutaneous neuropathy

Low back pain is one of the more common presenting neurologic complaints to a primary care provider. Most acute pain syndromes are benign, self-limiting conditions, with pain arising from myofascial sources. Patients with back pain and normal neurologic examinations are unlikely to have any serious underlying pathology and further diagnostic testing is usually unrevealing.

A 36-year-old auto mechanic presents to the emergency department after hurting his back on the job. While lifting an object, he experienced sudden pain in his lower back with radiation to the right buttock. He was initially treated for muscle strain with a nonsteroidal anti-inflammatory drug (NSAID) after x-rays of his lumbosacral spine demonstrated no pathology. He continued to complain of this low back pain now radiating posteriorly down his left leg to the mid-thigh. Physical examination is unremarkable. The most likely diagnosis is

Low back pain is one of the more common presenting neurologic complaints to a primary care provider. Most acute pain syndromes are benign, self-limiting conditions, with pain arising from myofascial sources. Patients with back pain and normal neurologic examinations are unlikely to have any serious underlying pathology and further diagnostic testing is usually unrevealing. (

complex regional pain

MC in wrist Sustained sympathetic activity in a perpetuated reflex arc characterized by pain out of proportion to physical exam findings also known as complex regional pain syndrome (CRPS) known as causalgia when associated with defined nerve Pathophysiology trauma from an exagerrated response to injury most common reason for a poor outcome following a crush injury to the foot surgery prolonged immobilization possible malingering Prevention vitamin C 500 mg daily x 50 days in distal radius fractures treated conservatively 200mg daily x 50 days if impaired renal function vitamin C also has been shown to decrease the incidence of CRPS (type I) following foot and ankle surgery avoid tight dressings and prolonged immobilization Prognosis responds poorly to conservative and surgical treatments Cardinal signs exaggerated pain swelling stiffness skin discoloration Physical exam vasomotor disturbance trophic skin changes hyperhidrosis "flamingo gait" if the knee is involved

A 60-year-old man accompanied by his daughter presents with a history of low back and buttock pain; it is 6/10 in severity. The pain starts when he stands or walks, but he can walk faster and with less intensity of pain when he is leaning over the shopping cart. On examination, the pain increases on extension of the spine. Question What is the best radiographic test for diagnosis and detailed evaluation of this condition?

MRI

Which type(s) of Salter-Harris fractures can generally be treated with closed reduction and cast immobilization?

Minimally displaced Salter-Harris types I, II, and III fractures generally can be treated with immobilization only. Types IV and V involve the cartilage of both the articular surface and the growth plate. To ensure proper alignment and a congruous joint surfaces open reduction and internal fixation is usually necessary

Which of the following patterns of stiffness is most characteristic of patients with rheumatoid arthritis? A morning stiffness lasting at least 1 hour B exacerbation of joint stiffness with walking C frequent, brief episodes of stiffness after inactivity D stiffness reflected by a major delay in muscle relaxation E stiffness evidenced by increased resistance to passive movement

Morning stiffness lasting at least 1 hour is characteristic of rheumatoid arthritis (RA). Exacerbation of joint stiffness with weight bearing (such as walking) and frequent, brief episodes of stiffness (lasting <30 minutes) after inactivity are both more characteristic of degenerative joint disease, not RA. Stiffness reflected by a major delay in relaxation after muscle contraction is seen in myotonic dystrophy. Stiffness evidenced by increased resistance to passive movement describes the "rigidity" associated with parkinsonism

hat is seen during the early stages of osteoarthritis?

Most descriptions of osteoarthritis emphasize the loss of articular cartilage that occurs as a result of this disease. However, during early stages of this disease, the cartilage is thicker than normal.

A 42-year-old woman works full-time as a data entry clerk and often puts in many hours of overtime. She has started to notice numbness and tingling in her right thumb, index finger, middle finger, and half of her ring finger; symptoms are especially severe at night The numbness and tingling were intermittent for months, but they have become persistent during the past few days. Question What could be considered a predisposing factor for this patient's condition?

Myxedema is one of the metabolic predisposing causes for carpal tunnel syndrome. The patient's clinical presentation is suggestive of carpal tunnel syndrome. Carpal tunnel syndrome is a median nerve compressive neuropathy. Patients with myxedema or hypothyroidism have accumulation of myxedematous tissue under the transverse carpal ligament, which causes compression of the median nerve in the carpal tunnel resulting in the manifestations of carpal tunnel syndrome. In carpal tunnel syndrome the median nerve is compressed in the wrist. The neurological distribution of her complaints corresponds to the distribution of the median nerve. The median nerve innervates the palmar surface of the thumb, index, middle, and radial half of the ring fingers and the motor branch innervates the thenar muscles. Electromyography and nerve conduction studies help in confirming the diagnosis of carpal tunnel syndrome

ankylosing spondylitis

NSAIDs, particularly indomethacin, are effective in controlling the symptoms of ankylosing spondylitis; NSAIDs are effective due to their anti-inflammatory properties as well as the relative safety of their long-term use.

A 50-year-old man was playing baseball with his company team last weekend and is now experiencing severe pain in his left shoulder. He states that he has been the team pitcher for several years now. He has developed pain progressively in his left shoulder over the last few months. He denies any numbness or tingling in his arm, hand, or fingers of his left upper extremity. Which exam would you expect to yield pain?

Neer test is the correct answer because this is the only exam listed that evaluates for a rotator cuff injury Bragard stretch test is incorrect because it evaluates for nerve root irritation at the level of L4, L5, and S1.

A 52-year-old overweight woman has had pain in her right hand for the past month. She is employed as a pastry chef and has trouble making a fist. On exam, she is tender over the radial styloid. You have her flex her thumb into her palm and move the wrist into ulnar deviation. This movement recreates her pain. What is the initial treatment for this condition?

Nonsteroidal anti-inflammatory drugs (NSAIDs) and thumb spica splint

A 16-year-old male soccer player is complaining of pain to the right foot that has been getting progressively worse for the last 2 months. He states it hurts the most when he has all of his weight on his right foot as he plants to kick the ball. Most of the pain appears to be on weight bearing. You are concerned that this patient may be developing a stress fracture. Based on the patient's history and patient presentations, which bone is the most affected by stress fractures in the foot?

Of the metatarsals, the second metatarsal has the highest number of stress fractures All types of fractures occur more easily in long thin bones like the metatarsals, than thicker bones like the calcaneus and talus

You are evaluating a 67-year-old female who has history and physical exam findings consistent with osteoarthritis. Based on your knowledge of the disease, which joints are most commonly affected? A Carpometacarpal (CMC) B Distal interphalangeal (DIP) C Metacarpophalangeal (MCP) D Proximal interphalangeal (PIP) E Radiocarpal

Osteoarthritis can affect all of the joints mentioned, but generally has the highest prevalence in the DIP joints, especially the second DIP joint. Osteophyte formation at the DIP joints produce enlargements referred to has Heberden's nodes.

40-year-old male carpenter presents with a 9-month history of worsening bilateral knee pain. He feels stiff in the morning when he first wakes up, but this subsides quickly. By the end of the workday, he has significant pain in his knees and has to go home and elevate his legs for some relief. This seems to be worse in colder weather. There is tenderness over the medial aspect of both knees, with the right knee being more tender and edematous. Varus deformity is noted. An arthrocentesis shows the synovial fluid white blood cell count to be 1200/mcL. On plain radiographs, subchondral cysts are present. Question What is the most likely diagnosis?

Osteoarthritis is correct because this often presents in a pauciarticular manner in middle-aged people. There can be morning stiffness, but it lasts less than 30 minutes. Osteoarthritis is worse with activity and is relieved by rest. On examination, there can be tenderness, crepitus, and joint effusion. Synovial fluid has less than <2000/mcL white blood cells. Plain radiographs can show subchondral cysts, osteophytes, subchondral sclerosis, and joint space narrowing.

What age group is most at risk to develop osteoid osteoma?

Osteoid osteoma is a benign bone forming tumor that usually develops during a patient's second decade of life

A 16-year-old girl presents with severe right knee pain; it started during a softball game 3 days ago after the patient swung to hit the ball. She reports hearing a popping sound before the pain began. There is no significant past medical history. Examination of the right knee reveals a positive 'apprehension sign'; Lachman, McMurray, valgus, and varus tests are negative. X-rays show a lateral tilt of the right patellar and subluxation. Question What is the most likely diagnosis?

Patellar dislocation is commonly seen with a history of twisting of the torso when the foot is planted. The positive apprehension sign, in conjunction with the X-ray findings, are diagnostic for this disorder only.

A 72-year-old female presents with a 4-month history of pain and stiffness in her shoulders and hips. She identifies the pain being worse in the morning and aggravated with getting in and out of the car along with difficulty brushing her hair. She also reports malaise and a 10-pound weight loss over the past few months. Her blood work shows an erythrocyte sedimentation rate (ESR) of 74 mm/h. What is the best treatment for the suspected diagnosis?

Patients with polymyalgia rheumatica (a clinical diagnosis based on pain and stiffness of the shoulder and pelvic girdle areas, frequently in association with fever, malaise, and weight loss) are treated with prednisone 10-20 mg/day orally. Usually after 2-4 weeks of treatment, slow tapering of the prednisone can be attempted. Most patients require some dose of prednisone for a minimum of approximately 1 year; 6 months is too short in most cases.

When considering the diagnosis of compartment syndrome, permanent damage to the muscle begins after how many hours of ischemia?

Permanent damage results after >8 hours (C) of ischemia. Nerves begin to lose conduction within 2 hours of onset of elevated pressures. Neurapraxia can occur within 4 hours, and irreversible damage occurs 8 hours after elevated pressures. Functional impairment is unlikely when compartment syndrome is diagnosed and treated within 6 hours of its onset.

A 52-year-old woman presents with left hip pain. There is no known history of trauma to the area. Her past medical history includes Crohn's disease, type II diabetes, and hypertension. She is currently taking metformin, metoprolol, and mesalamine; she recently completed a prednisone taper for her Crohn's disease. She also finished a course of ciprofloxacin, which she took for a urinary tract infection. An X-ray is obtained, and it reveals a collapsed left femoral head. Question What medication likely contributed to her current condition?

Prednisone is correct. The patient has a collapsed left femoral head on radiography, suggesting avascular necrosis. Avascular necrosis is commonly associated with the use of corticosteroids

A 12-year-old girl presents to the emergency room with worsening foot pain. 2 weeks ago, she stepped on a nail while wearing tennis shoes. The area was thoroughly cleaned, but she has developed worsening pain, redness, and drainage from the area. X-ray shows periosteal changes at the site of the wound. A wound culture is obtained. Question Which organism is most likely to grow on culture?

Pseudomonas aeruginosa is correct. Pseudomonas aeruginosa is the most common cause of osteomyelitis when a penetrating object passes through the sole of the shoe. Staphylococcus aureus is incorrect. While Staphylococcus aureus is the most common cause of osteomyelitis overall, Pseudomonas aeruginosa is the most common cause when the infection has been caused by a puncture wound through the sole of a shoe.

A 73-year-old man presents with inability to actively raise his left non-dominant arm to retrieve plates from the kitchen cabinet. This began a month ago after his shoulder pain improved. He had a history of pain in that shoulder for over 6 months that kept him from sleeping on the left side and the pain would wake him often. There was no specific injury he can recall although he felt a pop a month ago while taking out the trash. On inspection you notice the back of the shoulder appears sunken when compared to the other shoulder. Question What is the most likely diagnosis?

RTC tear

A 10-year-old boy presents with a possible arm fracture. While playing baseball at school, he slid into another student and hit his upper arm on the other student's leg. An X-ray of the patient's right arm reveals that he has a fracture at the distal 1/3 of the humerus. You also note that the patient cannot extend his wrist. at structure has most likely been injured?

Radial nerve

upper arm. He heard a pop and then felt sudden pain when it occurred. On exam, there is ecchymosis of the upper and middle arm; there is also a visible, palpable bulge proximal to the elbow. Although a relatively active and healthy patient for his age, he does have a longstanding history of impingement syndrome in the same arm. Question What treatment is indicated for this patient?

Range of motion and strengthening exercises are the best treatment for a ruptured proximal biceps tendon in an elderly patient as there is minimal loss of function. Surgical repair of the torn tendon is indicated in young, athletic patients.

A 12-year-old African-American boy presents with a 12-hour history of fever, chills, and knee pain. Plain radiograph is suspicious for osteomyelitis, and blood cultures are positive for Salmonellae. Question What is the patient's past medical history most likely to reveal?

Salmonellae has been identified as the most common cause of hematogenous osteomyelitis in the setting of hemoglobinopathies, such as sickle cell anemia. Diabetes mellitus is also associated with hematogenous osteomyelitis, but in these patients, the causative organism is commonly S. aureus, not salmonellae. Acute lymphoblastic leukemia, Osgood-Schlatter disease, and osteogenesis imperfect do not have a specific association with osteomyelitis.

A 28-year-old man presents first thing Monday morning due to a left ankle sprain. The patient describes walking in the woods the day before while wearing flip flops and inadvertently stepping into a hidden hole. The patient immediately noticed extreme pain with weight bearing on the left ankle, making it extremely difficult for him to walk out of the woods. Ambulation is still exceptionally difficult at this time. He describes it as an 8-9/10 on a 1-10 pain scale. The patient admits to having ankle sprains before, but this one is much more severe and debilitating. He has also noted significant swelling and exquisite tenderness to the touch. He has treated his ankle with elevation, ice for 20 minutes at a time, and ibuprofen 200 mg every 8 hours. Physical examination reveals a moderate degree of ecchymosis of the left ankle with substantial mechanical instability and moderate restriction of range of motion. Question Based on the history and physical examination findings, what would be the most appropriate clinical intervention at this time?

Subsequent treatment commonly involves protected weight bearing with crutches and the use of an ankle stabilizer brace. This is considered especially in those patient suspected of having either a Grade II or III ankle injuries.

A 4-year-old girl presents after a fall from a 4-foot slide 30 minutes ago. She fell with her right dominant arm outstretched and her elbow fully extended. There was no loss of consciousness, but there was extreme pain. She immediately grabbed her right forearm and her right elbow. There is an obvious deformity at the elbow. Question What is the most likely diagnosis?

Supracondylar fractures of the distal humerus are the most common elbow fractures in children between the ages of 2 and 12. The typical mechanism of this type of fracture is a fall on an outstretched arm, usually from a height.

Radiculopathy due to nerve root compression occurs most commonly at which nerve root within the brachial plexus?

The C7 nerve root is affected the most often (approximately 45-60%). This radiculopathy can result from foraminal encroachment of the spinal nerve, cervical disk herniation, tumor, and multiple sclerosis. C7 radiculopathy can present with weakness in the triceps, which cause elbow extension, and finger flexion and extension. C6 is another common site of radiculopathy. C6 radiculopathy can present with weakness in the biceps, brachioradialis, and wrist extensor muscles. Cervical radiculopathy at the C5, C8, and T1 are less common, but still possible. C5 radiculopathy can present with deltoid and biceps muscle weakness. C8 radiculopathy can present with finger flexor weakness and T1 radiculopathy with finger abduction weakness.

A 52-year-old male has a 200-pound file cabinet fall on his right leg. He comes to the emergency department complaining of pain and swelling to the right leg. He is also complaining of parasthesias to the leg also. Based on these findings on history, what part of the leg would be the most likely site of compartment syndrome? A Anterior compartment of the leg B Deep posterior compartment of the leg C Dorsal compartment of the forearm D Lateral compartment of the leg E Superficial posterior compartment of the leg

The Correct Answer is: A Anterior compartment syndrome is most commonly found in the anterior compartment of the leg, with the volar compartment of the forearm also a common location

A 23-year-old patient who has recently been on a ski trip presents with pain to the right hand after sustaining a fall. It is difficult to move, and there is pain on flexion of the digit. Based on this history what ligament would the patient most likely have injured? A 1st MCP joint ulnar collateral ligament B 2nd MCP joint ulnar collateral ligament C 3rd MCP joint ulnar collateral ligament D 4th MCP joint ulnar collateral ligament E 5th MCP joint ulnar collateral ligament

The Correct Answer is: A The ulnar collateral ligament at the base of the thumb, or 1st MCP joint, is often injured in forced abduction, such as a fall while skiing or during other sporting activities. An injury to this ligament has traditionally been called Gamekeeper's Thumb, but the origin of this term referred to a more chronic injury sustained by English gamekeepers as a result of the way they killed rabbits using their hands. Any of the MCP joint ulnar collateral ligaments could be injured in a fall if the mechanism of injury creates significant forces on the ligaments, but the 1st MCP joint is far more commonly injured than the others mentioned above.

A 35-year-old male presents with pain and decreased range of motion after sustaining a fall in which the patient tried to grab onto a bar which pulled his entire arm in the process. Given this clinical scenario, at what cervical motor neuron level would the biceps reflex be testing? A C4 B C5 C C6 D C7 E C8

The Correct Answer is: B C5 is the primary motor neuron being tested in a biceps reflex. C6 contributes to the brachioradialis reflex primarily, but does have a small role in the biceps reflex. The C7 motor neuron is primarily involved in the triceps reflex. C4 and C8 do not contribute to any primary reflexes.

When a bone is fractured, there are the 3 stages of healing. What is the proper order in which the healing occurs? A Inflammatory, Remodeling, Reparative B Inflammatory, Reparative, Remodeling C Remodeling, Inflammatory, Reparative D Remodeling, Reparative, Inflammatory E Reparative, Inflammatory, Remodeling

The Correct Answer is: B Inflammatory changes happen after fractures and this is followed by a reparative phase and ultimately, a remodeling phase. The healing from acute injuries generally starts with inflammation,. Once the integrity of the bone has been restored on the macro level, bone remodeling continues until full healing has occurred.

Upon testing a patient for function of the hip flexors, which muscle is considered the primary muscle responsible for most flexion? A Gracilis B Iliopsoas C Rectus femoris D Sartorius E Vastus intermedius

The Correct Answer is: B The iliopsoas muscle is the primary hip flexor muscle. It originates at T12 and L1-5 vertebrae and intervertebral disks as well as the iliac fossa of the pelvis and connects to the femur at the lesser trochanter. The gracilis muscle is considered a secondary hip adductor. Rectus femoris does help with hip flexion, but in a secondary role to the iliopsoas. Rectus femoris is also involved in knee extension. Sartorius is also involved in hip flexion, but in a secondary role. The vastus intermedius muscle is one of the four quadriceps muscles and is involved with knee extension and is not involved in hip flexion

On routine physical examination of a 24-year-old male, you note persistence of lumbar lordosis when the patient bends forward to touch his toes. There is no evident muscle spasm. Which of the following conditions best describes these findings? Ankylosing spondylitis Listing associated with herniated disc Scoliosis Spondylolisthesis

The correct answer is A. Ankylosing spondylitis leads to the loss of normal lumbar concavity and spine mobility. Lateral flexion is often the first movement lost. Palpation bilaterally over the sacroiliac joints may elicit tenderness. Disk herniation would be accompanied by radicular symptoms or perhaps sciatic nerve tenderness. Scoliosis will demonstrate a deviated spine and a thoracic deformity. With spondylolisthesis, one would expect to palpate a vertebral "step off," indicating the slippage of one vertebra upon another.

A 62-year-old male field hockey coach stopped to reverse his direction as his team's play transitioned from offense to defense. He felt a "pop" as his knee buckled and developed swelling. Aspiration of his knee yielded a hemarthrosis. You suspect a quadriceps tendon rupture. Which of the following exam maneuvers would be positive for a quadriceps tendon rupture? Straight-leg raise Lachman's test Valgus stress test McMurray's test

The correct answer is A. The straight-leg raise evaluates the competency of the quadriceps and patella tendon. If a patient is unable to perform a straight-leg elevation while lying in the supine position, the integrity of either extensor mechanism is in question. Lachman's test evaluates the anterior cruciate ligament (ACL) deficiency. With the thigh supported and thigh muscle relaxed, flex the knee to 25°. Grasp the distal femur from the lateral side with one hand and the proximal tibia from the medial side with the other hand. Initiate a shucking motion by pulling anteriorly on the tibia while pushing posteriorly on the femur. Focus on the amount of bony translation of the tibia relative to the femur. Increased anterior translation indicates a tear of the ACL. The valgus stress test evaluates the competency of the medial collateral ligament (MCL). Stabilizing the lateral lower thigh with one hand (above the knee), use the other hand placed on the inner lower leg to apply a valgus force onto the knee stressing the medial collateral ligament. If there is a laxity (or gapping) appreciated, one could clinically assess that there is a tear to the MCL. McMurray's test evaluates for meniscal tears. For a questionable medial meniscus tear, flex the knee of a patient who is lying in the supine position. With the knee flexed, externally rotate the tibia on the femur. Gradually extend the knee. If this maneuver elicits a click or pain, the clinician should be suspicious of a posterior horn medial meniscus tear.

Which of the following is false regarding fractures of the fifth metacarpal neck? It is caused by direct impact on the metacarpal head in a clenched-fist. It is commonly referred to as a Salter fracture. Palmar displacement is common. It is commonly referred to as a boxer's fracture.

The correct answer is B. A boxer's fracture is a fracture of the fifth metacarpal neck with volar displacement of the metacarpal head. It is typically caused by direct impact on the metacarpal head with the hand in a clenched fist position. The term Salter fracture describes a growth-plate injury in a child.

39. Which of the following is the most appropriate test for hip laxity and instability in an infant? Babinski response Barlow's test Galeazzi's test Ortolani's test

The correct answer is B. Barlow's test evaluates an intact hip for laxity and hip instability. With this test, the infant is positioned supine with the hips flexed to 90°. The clinician then adducts the hip and places downward pressure to sublux the hip posteriorly. An Ortolani maneuver also positions the patient supine with knees flexed 90°. The hip is then abducted (remember "out" for Ortolani), with upward pressure placed laterally on the greater trochanter (shown below). A positive Ortolani presents as a palpable clunk as the posteriorly dislocated hip relocates back into the acetabulum. Asymmetric skin folds may provide another clue to a unilateral hip dislocation. The left hip most often displaces because the most common fetal lie in utero adducts the left hip directly against the mother's lumbosacral spine. A positive Babinski response presents by dorsiflexion of the big toe and fanning of the other toes in response to plantar stimulation. It is a normal response from infancy up to age 2 years. The Galeazzi, or Alice, test checks for any variation in knee heights while the infant is supine with knees flexed. This indirectly assesses for a femoral length discrepancy.

A 69-year-old right-hand dominant diabetic patient presents complaining of an increasing flexor contracture of his right ring finger over the past 6 months, as shown below. Which of the following is the most likely diagnosis? Save Image Enlarge Image Boutonnière deformity Dupuytren's contracture Swan neck deformity Trigger finger

The correct answer is B. Dupuytren's contracture results from a progressive plaque formation along the flexor tendon, which ultimately results in the formation of a fibrous cord. Boutonnière and swan-neck deformities frequently occur in chronic rheumatoid arthritis. Trigger finger is caused by a nodule at the head of the metacarpals, which makes the finger get "caught" in the flexed position and "pop" into extension.

A 19-year-old sophomore on the local university football team sustained an impact injury from an opponent's helmet to his anterior lower leg. He continued playing and finished the game. He has since developed increasing pain and now presents at the emergency department. The radiographs are normal. On physical examination, passive plantar flexion of the ankle provokes extreme pain that is disproportionate to the injury. He also has some sensory hypoesthesia. Which of the following should be your most pressing concern? Occult tibia fracture Achilles tendon rupture Compartment syndrome Deep vein thrombosis

The correct answer is C. A compartment syndrome involves increased interstitial pressure within a closed fascial compartment. It can obstruct microcirculation to the nerves and muscles lying within the involved space, causing tissue necrosis that usually becomes irreversible after 4 to 6 hours. The term occult fracture describes a suspected fracture that is not visible on radiograph (e.g., stress fractures or fatigue fractures). The Achilles tendon is a band of tissue that connects the heel bone to the calf muscle. Tear of this tendon would cause obvious functional deficits to the patient's plantar flexion capability. A torn Achilles tendon will result in a positive Thompson's test. The formation of a blood clot within one or more veins in the deep venous system of the upper or lower extremities refers to a deep vein thrombosis.

Which of the following is false regarding the innervation of the hand? The median nerve innervates the thenar eminence for both sensory and motor functions. The patient should abduct and adduct fingers against resistance to check the function of the radial nerve. The radial nerve innervates much of the skin on the dorsum of the hand. The ulnar nerve innervates muscles on the small-finger side of the hand, which is responsible for the majority of the strength utilized when an object is grabbed.

The correct answer is B. The nerve supply to the hand is complex. The ulnar nerve is responsible for abduction and adduction of the fingers. The radial nerve is responsible for wrist extension and thumb extension. The radial nerve does provide sensation to much of the skin on the dorsum of the hand. The median nerve clearly provides the sensory and motor function to the thenar eminence.

33. A 14-year-old boy fell skateboarding. He has pain in his elbow. His radiograph is shown below. How would you describe this injury? Save Image Enlarge Image Greenstick fracture Occult fracture Fatigue fracture Pathologic fracture

The correct answer is B. The term occult fracture is sometimes used to refer to a fracture that does not appear on radiograph (e.g., a stress fracture or a fatigue fracture). A greenstick fracture is an incomplete fracture in a bone of a child. With a greenstick fracture, one cortex is broken and the other cortex and periosteum remain intact on the compression side of the fracture. A fatigue fracture is a fine hairlike fracture that develops from repetitive microtrauma, such as distance running. A pathologic fracture is an occurrence of a fracture secondary to underlying pathology (e.g., metastatic bone lesion).

Which of the following is the least important part of the neurologic examination in patients with low back pain? Evaluate strength in ankle plantar flexors to test motor function at the S1/S2 level. Evaluate the popliteal reflex to test L5. Evaluate the Achilles reflex to test S1. Evaluate the extensor hallucis longus or (great toe dorsiflexor) to test L5.

The correct answer is B. There is no popliteal reflex, and all of the other answers represent important parts of the neurologic exam in a patient with low back pain.

A patient who complains of left shoulder pain rotates her forearm medially against resistance (shown below). Which of the following rotator cuff muscles does this physical maneuver best assess? Save Image Enlarge Image Infraspinatus Subscapularis Supraspinatus Teres minor

The correct answer is B. To assess for subscapularis integrity, the patient rotates the forearm medially against resistance. Having the patient abduct the forearm against resistance assesses the supraspinatus. When the patient rotates the forearm laterally against resistance, then the clinician evaluates the infraspinatus and teres minor muscles.

32. A healthy, athletic 21-year-old female falls while skiing. She sustained a compound fracture of her fifth metacarpal. You evaluate her in the emergency department, and the fracture is reducible. She states she has minimal pain. Which of the following would be most appropriate as her initial treatment? Ulnar gutter splint with referral to orthopedic office Short arm cast with follow-up for x-ray in 1 week to confirm fracture position Emergent open irrigation of wound site and antibiotics Short arm cast with immediate referral to orthopedic office

The correct answer is C. A compound fracture is an open fracture. Any break in the skin that communicates with the hematoma and injured tissue surrounding a fracture permits bacterial contamination of the wound. Prompt surgical debridement is required. The patient should also receive antibiotics and a tetanus update. None of the other answers is acceptable.

Which of the following best describes the fracture shown in the radiograph below? Save Image Enlarge Image Fatigue Impacted Torus Epiphyseal

The correct answer is C. A torus fracture is a compression fracture of the cortex occurring 2 to 3 cm proximal to the physis. It is most common in the distal radius of a child. A fatigue fracture is a fine hairlike fracture that develops from repetitive microtrauma, such as distance running. The term impacted fracture may be applied to a fracture in which the ends of the bones are wedged together (and shortened). An epiphyseal fracture is a growth-plate fracture. It can be classified according to an injury pattern (Salter-Harris classification).

36. Which of the following is consistent with a sensorineural hearing loss? The patient's hearing seems to improve in noisy surroundings. It may result from otosclerosis of the ossicles. With Rinne testing, air conduction is greater than bone conduction. With Weber testing, the sound lateralizes to the impaired ear.

The correct answer is C. Answers A, B, and D occur in conductive hearing loss. With a sensorineuronal hearing loss, the sound would lateralize to the unaffected ear on Weber testing (shown below, see first image), and air conduction would be greater than bone conduction on Rinne testing (shown below, see second two images).

An 82-year-old female presents to discuss progressive motion deficits and pain in her "arthritic hands." She has osteoarthritis, and visually you notice deformity and swelling of the distal interphalangeal joints of her fingers, as shown below. Which of the following best describes these nodules? Save Image Enlarge Image Bouchard's nodes Heinz nodes Heberden's nodes Boutonniere nodes

The correct answer is C. Heberden's nodes are hard nodules or enlargements of the tubercles of the distal interphalangeal joints typically associated with osteoarthritis. Bouchard's nodes are bony enlargements or nodules located at the proximal interphalangeal joints; they are typically associated with rheumatoid arthritis. Heinz nodes are a fictitious diagnosis (although a Heinz body is a granule in red blood cells causing damage to the hemoglobin molecules). Boutonnière nodes are another fictitious diagnosis (although Boutonnière deformity is a contracture of hand musculature marked by PIP joint flexion and DIP joint hyperextension).

An 82-year-old woman falls on the ice. Emergency medical services (EMS) transports her to the emergency department. On physical exam, she has a shortened, externally rotated left leg and has increased pain in the same hip. She is somewhat more comfortable when lying quietly, but screams in pain with any movement. She is very tender to palpation over her proximal thigh and has swelling in that area. The radiograph is shown below. Which of the following is the most likely diagnosis? Hip dislocation Distal femoral fracture Hip fracture Pelvic fracture

The correct answer is C. Hip fracture is the most likely diagnosis. Patients present with the leg in an externally rotated and shortened fashion. Hip dislocations are much less common. Pelvic fractures do not cause shortening of the limb, and the other problems are present at the knee, not the hip.

A 3-year-old girl presents to the pediatric office where her parents brought her. She was playing with her older brother today, and he was swinging her around by her arms. Immediately afterward, she began to cry and stopped using her right arm. On physical exam, she sits quietly and holds her right arm with the elbow extended and the forearm pronated. She has no swelling or ecchymoses, but refuses to move her right elbow. You obtain a radiograph of the right elbow and it is negative for fracture. Which of the following is the most likely diagnosis? Radial head fracture Distal humeral fracture Subluxation of the radial head (nursemaid's elbow) Ulnar shaft fracture

The correct answer is C. Nursemaid's elbow is the most common elbow injury in young children. Treatment almost always includes closed reduction in the office or emergency department. All of the other injuries would probably present with swelling. All of the fractures would have radiographic evidence of injury.

36. A 39-year-old patient has a past history of a shoulder dislocation. He experiences discomfort in his shoulder with external rotation maneuvers of his right arm (e.g., taking off his glasses at night and putting them on the nightstand while lying in bed). You are concerned about his shoulder stability. Which of the following clinical tests best describes placing the patient's arm at 90° abduction and then externally rotating his shoulder? Sulcus sign Jerk test Apprehension sign Impingement sign

The correct answer is C. The apprehension sign evaluates for chronic shoulder dislocation (instability). Please note the picture below. The clinician should abduct and externally rotate the patient's arm to a position where it might easily dislocate. If the patient is experiencing impending dislocation, the patient will have a noticeable look of apprehension and will resist further motion. To perform this test, the examiner should place the patient's arm at 90° abduction and then externally rotate the shoulder. The sulcus sign tests for inferior shoulder laxity. The clinician should apply traction in an inferior direction with the arm relaxed at the patient's side. This maneuver causes inferior subluxation of the humeral head and a widening of the sulcus between the humerus and the acromion in a person with inferior shoulder laxity. The jerk test evaluates posterior capsule instability. The patient's arm is placed in 90° of flexion and maximum internal rotation with the elbow flexed at 90°. The arm is then adducted across the body in the horizontal plane while pushing the humerus in a posterior direction. The test is positive if a posterior subluxation or dislocation occurs. The impingement sign is as follows: With the patient seated, depress the scapula with one hand while elevating the arm with the other. This maneuver compresses the greater tuberosity against the anterior acromion and elicits discomfort in patients who have impingement syndrome.

A 25-year-old woman falls on an icy sidewalk and lands on her right hand. She immediately feels pain in her elbow but denies other injuries. On physical exam, she has swelling around the elbow joint and cannot extend or supinate her elbow. The radiographs are shown below. Which of the following is the most likely diagnosis? Olecranon bursitis Anterior shoulder dislocation Radial head fracture Distal radial fracture

The correct answer is C. The most common fracture at the elbow is a radial head fracture. Commonly, the patient has tenderness and pain and is unable to fully extend or supinate the elbow. Olecranon bursitis is commonly confused with cellulitis and is not a fracture.

A 19-year-old boy falls onto his outstretched hand while playing basketball. He has pain in the anatomical snuffbox upon palpation. His initial x-rays are noncontributory. Which of the following is not appropriate management of this patient? Immediate immobilization by thumb spica splint or casting. Repeat radiographs or perform an MRI in 10 to 14 days if the clinical exam is still suggestive of a fracture. Radiographs should include scaphoid or navicular views initially. If initial wrist x-rays are negative, the patient may return to sports in 48 hours.

The correct answer is D. Anatomic snuffbox tenderness after a fall is considered a fracture until proven otherwise. The scaphoid is the most commonly fractured carpal bone, and initial radiographs can prove to be negative up to 15% of the time. The clinician should take care to obtain navicular or scaphoid views of the patient's wrist. Immediate immobilization and referral to an orthopedic specialist is the most appropriate management. Repeat radiographs, an MRI, or a bone scan are all possible radiographic options for these patients on follow-up.

A 35-year-old skier is referred to your office. She has pain in her knee after twisting it while skiing. She thinks that she heard a "pop" and then felt pain and swelling. On physical exam, she has a knee effusion and laxity on Lachman's test as well as laxity on the anterior drawer test. Which of the following is the most likely diagnosis? Lateral collateral ligament injury Patellar tendon rupture Chondromalacia patella Anterior cruciate ligament injury

The correct answer is D. Anterior cruciate injuries are common. The patients often are young, and the injuries frequently occur as a result of twisting, rotational injuries. Patients usually develop an immediate joint effusion and have a positive Lachman's test along with laxity on the anterior drawer test.

Which of the following cranial nerves does the exam technique shown below assess? Save Image Enlarge Image V (trigeminal) VII (facial) X (vagus) XI (spinal accessory)

The correct answer is D. Cranial nerve XI (spinal accessory nerve) innervates both the sternocleidomastoid and the trapezius muscles. Turning the head against resistance evaluates the motor strength of the sternocleidomastoid muscles; shrugging the shoulders against resistance tests the strength of the trapezii. Palpating the patient's temporal and masseter muscles while the patient clenches his or her teeth evaluates cranial nerve V (trigeminal nerve). Maneuvers that assess cranial nerve VII (facial nerve) include asking the patient to: raise both eyebrows, frown, smile, show upper and lower teeth, puff out the cheeks, and to keep eyes tightly closed while the examiner attempts to open them. Asking the patient to say "ahh" and watching for the symmetric rise of the uvula best evaluates cranial nerve X (vagus nerve).

Which of the following is the name of the maneuver, shown below, that is used to test for DeQuervain's tenosynovitis? Save Image Enlarge Image Allen's test Tinel's test Phalen's test Finkelstein's test

The correct answer is D. Finkelstein's test evaluates for De Quervain's tenosynovitis of the wrist. The practitioner flexes and ulnarly deviates the wrist while the thumb is grasped into flexion. Pain at the dorsoradial aspect of the wrist indicates tenosynovitis of the first dorsal compartment (abductor pollicis longus and extensor brevis) tendons. Allen's test evaluates blood supply to the hand. The patient first opens and closes his fist several times. With the patient's fist closed, the clinician applies pressure to the radial and ulnar arteries in order to occlude them. When the patient opens his or her hand, the examiner releases pressure from one of the arteries and the hand should flush immediately. If the hand does not flush or reacts slowly, then the assumption is that one of the arteries is partially or completely occluded. Phalen's test evaluates carpal tunnel syndrome. The patient holds both forearms perpendicular to the ground, then flexes both wrists, while holding them against each other for 30 seconds. Patients who experience numbness or tingling in the distribution of the median nerve have a positive result. Tinel's sign also evaluates carpal tunnel syndrome. A cutaneous tingling sensation occurs when the clinician taps directly on the damaged nerve trunk (over the volar retinaculum).

Which of the following is not a key part of the diagnosis of a patient with cauda equina syndrome? Bladder dysfunction (usually overflow incontinence) Perineal numbness or numbness in the inner thighs (saddle paresthesias) Bilateral radiating leg pain below the knees Hip pain

The correct answer is D. Hip pain is not necessarily a part of cauda equina syndrome. All of the other answers are clearly part of the syndrome. It is important to identify cauda equina syndrome, because it is a surgical emergency.

Which of the following sensations would most likely be lost or impaired in patients with peripheral neuropathy? Graphesthesia Stereognosis Two-point discrimination Vibration

The correct answer is D. Loss of vibratory sense is usually the earliest indication of peripheral neuropathy. Impaired graphesthesia, stereognosis, and two-point discrimination suggest a lesion in the sensory cortex. Graphesthesia translates from Greek as "writing perception" and is the patient's ability to identify a character traced on the palm of his or her hand. Stereognosis refers to the patient's ability to recognize objects placed in his or her hand solely by touch while the eyes are closed.

A 32-year-old male playing tennis lunged for a ball going down the sideline. Upon doing so, he felt a sudden pain in the back of his leg. Which of the following positive physical examination findings would be consistent with his injury? Thomas' test Homans' test Tailor's test Thompson's test

The correct answer is D. This patient has probably ruptured his Achilles tendon. Thompson's test evaluates the integrity of the Achilles tendon. To do this test, the patient hangs his or her lower legs off the end of the exam table while lying in the prone position. The examiner squeezes the calf while observing for passive plantar flexion. The result is positive if there is no movement of the foot. This would indicate an Achilles tendon rupture. Homans' test is an early sign indicating venous thrombosis of the deep veins of the calf. Current literature suggests that performing a Homans' test may actually cause the clot to dislodge and place the patient at increased risk, and several texts recommend against performing this test. Diagnosis of deep vein thrombosis bases primarily on physical exam, Doppler ultrasound, and a positive D-dimer. When the foot is passively dorsiflexed, the patient has pain produced in the calf. Tailor's test is positive when pain is elicited along the sartorius muscle of the thigh as one actively flexes, abducts, and laterally rotates the thigh. This occurs with an injury to the sartorius muscle. The sartorius muscle is the longest in the body and enables the crossing of the legs in the "tailor's position." Thomas' test is used to check for flexion contracture of the hip. The clinician flexes the patient's hip as the patient lays in the supine position. This causes the lumbar spine to flatten, which stabilizes the pelvis. Further flexion can then only originate in the hip joint. The normal limit for hip flexion is approximately 135°. Inability to extend the leg straight without arching the thoracic spine constitutes a fixed flexion contracture. The extent of a flexion contracture can be determined by estimating the angle between the table and the patient's leg.

A 47-year-old man presents with a history of worsening burning left heel pain for the last few weeks. He states that it is worse in the morning after getting out of bed and beginning to walk; it improves with exercise. He denies any foot numbness, tingling, or known injury. He states that he recently began a jogging regimen. Question What is the most likely diagnosis?

The correct answer is plantar fasciitis, as it occurs as a result of excess stress on the Achilles tendon that attaches to the plantar fascia. Classic presentation is burning heel pain, which is worse when bearing weight after prolonged periods of rest (e.g., sleep). Heel contusion is a soft tissue injury resulting from a trauma. Heel contusions cause pain, but the pain is unlikely to be as severe as what is described in this case. Calcaneal fracture typically occurs after an injury, and the pain would likely be more consistent with weight bearing. Tarsal tunnel syndrome is due to entrapment of the posterior tibial nerve, and it presents with medial malleolus pain that may radiate. Pain is typically worse during sleep, walking, and dorsiflexion. Paresthesias are common. Achilles tendon rupture is often described as the feeling of being kicked or shot in the area of the Achilles tendon; subsequently, jogging and walking normally would not be possible, as the tendon is crucial for this movement.

A 68-year-old man presents with progressive onset of lower back pain, unsteady gait, and numbness and occasional weakness in his lower extremities. His symptoms seem worse after a day at work, where he stands for hours at a time. He finds that resting, especially sitting, helps alleviate the symptoms. what first-line management should be discussed with this patient?

The correct answer is to implement a regular exercise program. Patients with diffuse spinal canal narrowing have what is known as spinal stenosis and are not restricted in their activity, but they often limit their own activity due to intolerance and pain.

40. A 51-year-old female presents with foot pain that improves when she removes her shoes. The physical exam reveals pain between the third and fourth toes. Her pain increases when squeezing the metatarsal heads together with medial/lateral compression. Which of the following is the most likely clinical diagnosis? Plantar callus Plantar fasciitis Hammer toe Morton neuroma

The correct answer is D. Morton neuroma is a perineural fibrosis of the common digital nerve as it passes between the metatarsal heads. The fibrosis is secondary to repetitive irritation of the nerve. The condition is most common between the third and fourth toes (web space). It has a female to male ratio of 5:1. A plantar callus is hardened skin located at the bottom of the foot, typically as a response to excessive pressure over a bony prominence. Plantar fasciitis is the most common cause of heel pain in adults. The etiology is probably a degenerative tear of part of the fascial origin from the calcaneus, followed by a tendinosis-type reaction. It affects women twice as often then men and is more common in overweight persons. A hammer toe is a flexion deformity of the proximal interphalangeal (PIP) joint with no significant deformity of the distal interphalangeal (DIP) or metacarpophalangeal (MP) joints. A callus (corn) can develop on the dorsum of the PIP joint due to increased pressure and demonstrates accentuation of the skin lines.

A 13-year-old soccer player presents complaining of bilateral knee pain that worsens when he plays soccer. He notes that the pain seems to feel better after resting for a day or two. On physical exam, he has tenderness and swelling over both knees at the level of the tibial tuberosities. Which of the following is the most likely etiology of his knee pain? Chondromalacia patella Bilateral tibial stress fractures Cruciate ligament injuries Osgood-Schlatter disease

The correct answer is D. Osgood-Schlatter disease is common in adolescents. They often present with symptoms after running, jumping, or squatting activities. All of the other diagnoses are rarely bilateral, except for chondromalacia, which does not cause tenderness and swelling at the tibial tubercles.

In nontraumatic back pain, "red flags" in a patient's history or physical might lead the clinician to believe that the back pain is something more than just a musculoskeletal problem. Which of the following patients with low back pain is least likely to raise a red flag? A 25-year-old female IV drug abuser with back pain and a fever An 11-year-old with back pain and no apparent injury A 40-year-old woman with a history of ovarian cancer A 35-year-old construction worker with a complaint of back pain when lifting at work

The correct answer is D. Patients older than 50 and younger than 18 having complaints of back pain are cause for concern. A history of ovarian cancer should raise concern about metastatic disease. Back pain in an IV drug abuser with a temperature should also cause concern about an infectious cause.

Which of the following terms best describe an acutely painful, erythematous great toe that is warm to the touch accompanied by a recently elevated uric acid level? Pogoniasis Podobromidrosis Tophyperidrosis Podagra

The correct answer is D. Podagra, more commonly referred to as gout, most commonly affects the joints of the foot or of the great toe. Pogoniasis is excessive growth of the beard or growth of a beard in a woman. Podobromidrosis is offensive perspiration of the feet. Tophyperidrosis is excessive sweating in local areas.

35. A 7-year-old girl has fallen off the monkey bars. She sustained a fracture of her distal radius that involves the growth plate (epiphysis). On radiograph, the fracture runs through the epiphyseal line. Which Salter classification would this represent? I II III IV

The correct answer is D. Salter classification: Salter I: Fracture at the level of the growth plate Salter II: Fracture above the level of the growth plate Salter III: Fracture below the level of the growth plate Salter IV: Fracture that traverses through the growth plate from the metaphysis to the epiphysis Salter V: Fracture that crushes the growth plate

Which of the following sensory discriminatory functions is impaired in a patient who cannot properly identify an item such as a key when it is placed in his or her hand? Extinction Graphesthesia Proprioception Stereognosis

The correct answer is D. This best describes stereognosis. With extinction, the patient (with closed eyes) identifies places the examiner touches, pointing to the locations of simultaneous bilateral stimuli. Graphesthesia is the technique whereby a patient identifies a number or letter traced on the palm of his or her hand. Propioception evaluates body positioning and coordination and is a good reflection of a patient's cerebellar function

A 25-year-old man presents with back pain and stiffness. He states that he has had longstanding issues with back pain, although he denies any trauma to his back. He has noticed, along with the pain, an increasing presence of stiffness and general fatigue. He feels that these issues have gradually worsened over the last several months, and they are more persistent recently. The pain is much worse first thing in the morning, and he rates it a 7/10 on a numerical pain scale. Radiation occasionally occurs into the buttock areas bilaterally and the patient feels the symptoms markedly lessen with activity. Physical examination shows marked forward stooping of the thoracic and cervical spine, with the lower spine showing presence of a substantial reduction in lateral flexion. Question What is high on your differential diagnosis?

The patient history and physical examination being described above is most likely a case of ankylosing spondylitis (AS). This is a chronic inflammatory disease that consists of many signs of symptoms, specifically significant back pain and progressive spinal stiffness. Many times patients with AS will also have transient or persistent peripheral arthritis as well as other manifestations such as anterior uveitis. Typically this is seen in male patients who are in the age range of 20-30 years old, although some may start having issues as early as their late teens. and minimize extraspinal and extraarticular manifestations and comorbidities. Spondylolisthesis is described as a forward or backward displacement of a single or multiple vertebrae. Patients with this condition may very well not have any symptoms, but if signs or symptoms are present, they could include walking abnormalities, postural problems, constant pain in the lumbar region, and (especially) leg pain. Often, this pain is worsened with activity and relieved with rest. Osteoarthritis is not as likely due to the fact that this is generally seen as a disease of aging; 90% of patients over the age of 40 years old have some degree of radiologic evidence of osteoarthritis. It also affects more women than men. Symptoms and signs usually affect the PIP joints, DIP joints, carpometacarpal joint of the thumb, the hip, knee, or secondary in origin to a joint that has other injuries (mechanical overuse, metabolic disease, or mechanical injury.) Onset is insidious. There is stiffness of 15 minute duration, with pain made worse with activity or weight bearing, and it is relieved by rest. Flexion contracture may be evident as well as potentially Bouchard or Heberden nodes. No ankylosis is present, but there may be limitation of the joint, crepitus, or effusion may be evident. Rheumatoid arthritis is not as likely in this case scenario; ithas an insidious onset with characteristic morning stiffness and symmetrically affecting small joints of the hands and feet. Psoriatic arthritis also is not consistent with the patient scenario above due to the fact that psoriasis usually precedes the arthritis component is around 80% of the cases. This form of arthritis affects the DIP and SI joints. Usually, this is asymmetric, and it may present with a classic 'sausage' appearance of the affected digits.

Compartment Syndrome

The patient is suffering from compartment syndrome. Compartment syndrome occurs when pressure increases inside of a muscle compartment. It typically develops within a few hours following soft tissue injury. Pain with extension of the involved limb is the most reliable early clinical finding. Extension of the involved limb will increase the pressure within the involved compartment, thereby increasing pain. Pain is typically out of proportion to exam findings and is unresponsive to opioid analgesia

Osteomyelitis Abx Tx

The patient will need to be on antibiotics for at least 6 weeks. It is highly likely that the patient above has an acute case of osteomyelitis. In simplest terms this is an infection of the bone involved.

A patient complains of pain during medial rotation of the humerus but not during abduction or extension of the shoulder. What muscle is injured?

The pectoralis major flexes, adducts, medially rotates, and is a minor extensor of the shoulder. The teres minor is a lateral rotator. The latissimus dorsi extends and medially rotates. The deltoid abducts and can assist in flexion, extension, and medial and lateral rotation. Only the subscapularis medially rotates and is not involved in extension and abduction. Therefore, this is the injured muscle.

Upon testing a patient for function of the hip abductors, which muscle is considered the primary muscle responsible for most abduction? A Biceps femoris B Gluteus maximus C Gluteus medius D Gluteus minimus E Vastus medialis

The primary mover in the motion of hip abduction is the gluteus medius muscle. Gluteus minimus does play a supporting role in that motion. Biceps femoris is one of the three hamstring muscles and contributes to the motions of knee flexion (primary muscle) and hip extension (secondary muscle). Gluteus maximus is the primary mover for hip extension and vastus medialis is one of the four quadriceps muscles responsible for knee extension, but no hip movements. (

A 45-year-old woman with recent diagnosis of rheumatoid arthritis has begun treatment with celecoxib. She has been on this medication for 3 months and notes that her pain continues. Early signs of joint involvement are present in the patient's hands. Which of the following medications is the most appropriate to add to her treatment?

The treatment of rheumatoid arthritis (RA) is aimed at reduction of pain, preservation of function, and prevention of deformity. Although non-steroidal anti-inflammatory drugs (NSAIDs) provide symptomatic relief, they do not alter progression or prevent erosion of the joint. Consequently, in addition to NSAID therapy, disease-modifying anti-rheumatological drugs (DMARDs) should also be initiated as soon as the diagnosis is confirmed.

A 3-year-old boy has velvety lax skin, hyperextensible joints, and mitral valve prolapse. What is the most likely diagnosis?

There are several different types of Ehlers-Danlos syndrome (EDS).

Inflammatory arthropathies

These include ankylosing spondylitis and psoriatic arthritis. Spondylitis is characterised by thinning or loss of elasticity of the discs that cushion the vertebrae of the spine, and is the most common cause of chronic low back pain in the middle-aged and elderly. It is more common in men and tends to run in families. Patients commonly have marked stiffness on awakening and pain that alternates from side to side of the lumbar spine. Exercise relieves the pain but pain is made worse by bending, lifting and prolonged sitting in one position (e.g. long car journeys).

Spinal Cord Tumors

This patient has an ependymoma. Ependymomas make up approximately 60% of spinal cord tumors. They are relatively slow-growing and may develop anywhere along the entire length of the spinal cord. However, a large percentage of these tumors are found at the lower sections of the cord. They develop from ependymal cells and can be seen in all age groups; however, the most common age group is between 20 and 40 years. Men and women are affected equally. Tumors may metastasize here from other parts of the body or they may be made of the same types of cells. Spinal tumors may be intradural, extramedullary, intramedullary, or extradural. Astrocytomas are the 2nd most common type of spinal cord tumor. They are found more often in children than in adults

A 42-year-old woman works full-time as a data entry clerk and often puts in many hours of overtime. She is experiencing numbness and tingling in her right thumb, index finger, middle finger, and half of her ring finger. The numbness and tingling initially comes and goes; however, after a few months, it is constantly present. Question What nerve is involved?

This patient has symptoms consistent with carpal tunnel syndrome. In carpal tunnel syndrome, the median nerve is compressed in the wrist. The distribution of her symptoms indicates distribution of the median nerve. The median nerve innervates the skin on the lateral 2/3 of the hand and fingers on the palmar side and the dorsum of digits 2, 3 and some of 4. The median nerve innervates most of the muscles of the anterior forearm and some of the muscles of the hand. The ulnar nerve innervates the skin of the medial third of the hand.

A 13-year-old boy presents with a 5-month history of intermittent right knee pain. He notes that his pain is poorly localized over the anterior knee, but it is provoked by activities such as running, jumping, squatting, and climbing or descending stairs. As a result, he has had to curtail recreational games at recess and home. He denies any history of trauma, falls, injury, fever, chills, swelling, skin changes, hip pain, or ankle pain. His physical exam reveals a boy with an appropriate build. His right knee is notable for point tenderness over an enlarged and prominent tibial tubercle. There is associated focal swelling. An in-office X-ray is performed, demonstrating the following image. Question What is the most appropriate pharmacotherapeutic agent at this time?

This patient is presenting with signs and symptoms consistent with Osgood-Schlatter disease. Therapy for Osgood-Schlatter disease (OSD) is conservative. Initial treatment includes the application of ice for 20 minutes every 2 - 4 hours. The only medications that need to be prescribed are nonsteroidal anti-inflammatory drugs (NSAIDs), such as oral ibuprofen, for pain relief and reduction of local inflammation (any NSAID may be used). Along with nonsteroidal anti-inflammatory drugs (NSAIDs), analgesics (e.g., acetaminophen) may be given for pain relief and reduction of local inflammation.

The patient has osteomyelitis. Staphylococcus auerus is the most common cause of osteomyelitis of the foot in diabetic patients.

This patient likely has a fracture of the olecranon. With this fracture, injury to the ulnar nerve is most possible. The ulnar nerve supplies sensation of the little finger palmar surface. Numbness of the dorsal thumb and back of the hand would be an injury to the radial nerve. Vascular compromise is rare with this injury, and thus capillary refill and radial pulse should be normal

A 52-year-old man with hypertension associated with recent unexplained weight loss presents with fever, malaise, and gradual onset of pain and weakness of his leg muscles for the past month. Physical examination reveals a mottled reticular pattern overlying portions of both calves and an area of ulceration with surrounding induration on the left lateral malleolus. Initial laboratory results reveal mild normochromic anemia, leukocytosis, and elevation of C-reactive protein, BUN, and creatinine. Which of the following is the most appropriate diagnostic evaluation to confirm the suspected diagnosis?

This patient most likely has polyarteritis nodosa (PN). A major obstacle in making the diagnosis is the absence of a disease-specific serological test. The diagnosis requires confirmation with either a tissue biopsy or angiogram.

A 52-year-old man with hypertension associated with recent unexplained weight loss presents with fever, malaise, and gradual onset of pain and weakness of his leg muscles for the past month. Physical examination reveals a mottled reticular pattern overlying portions of both calves and an area of ulceration with surrounding induration on the left lateral malleolus. Initial laboratory results reveal mild normochromic anemia, leukocytosis, and elevation of C-reactive protein, BUN, and creatinine. Which of the following is the most appropriate diagnostic evaluation to confirm the suspected diagnosis?

This patient most likely has polyarteritis nodosa (PN). A major obstacle in making the diagnosis is the absence of a disease-specific serological test. The diagnosis requires confirmation with either a tissue biopsy or angiogram.

A 59-year-old woman with a known history of rheumatoid arthritis presents with relatively severe complaints of pain, notable bony deformity of the hands with extra-articular findings of cutaneous nodules, scleritis, and pleurisy. On physical examination, the patient is found to have splenomegaly. Which of the following is the most appropriate laboratory evaluation to order to further evaluate the suspected diagnosis?

This patient presentation of known rheumatoid arthritis with severe deformities, extra-articular findings, and splenomegaly is most likely Felty syndrome. Felty syndrome is characterized by the triad of deforming rheumatoid arthritis, splenomegaly, and neutropenia. The appropriate laboratory test to order would be a CBC to evaluate for neutropenia.

A 59-year-old woman with a known history of rheumatoid arthritis presents with relatively severe complaints of pain, notable bony deformity of the hands with extra-articular findings of cutaneous nodules, scleritis, and pleurisy. On physical examination, the patient is found to have splenomegaly. Which of the following is the most appropriate laboratory evaluation to order to further evaluate the suspected diagnosis?

This patient presentation of known rheumatoid arthritis with severe deformities, extra-articular findings, and splenomegaly is most likely Felty syndrome. Felty syndrome is characterized by the triad of deforming rheumatoid arthritis, splenomegaly, and neutropenia. The appropriate laboratory test to order would be a CBC to evaluate for neutropenia. Uric acid testing is helpful in evaluating gout but is not relevant to this patient presentation. Ordering an erythrocyte sedimentation rate or C-reactive protein is not necessarily helpful in diagnosing Felty syndrome; in an acute inflammatory flare, both would most likely be elevated. Antinuclear antibodies could be present in 20% to 40% of patients but are not diagnostic of Felty syndrome.

A 35-year-old man with no significant past medical history has been experiencing progressive, moderately severe right knee pain for the past 3 months. Pain was initially felt only at night; however, it is now constant throughout the course of the day for the last several weeks. It is especially severe upon ambulation and during knee ranges of motion, causing him to limp. He denies fever, chills, weight changes, history of gout, sexually transmitted diseases, hip or back pain, recent instrumentation, trauma, or injuries. His physical exam reveals an antalgic gait with limp, limited ranges of motion of the right knee, and a 3 cm diameter firm, tender mass at the distal femur. There is no erythema, crepitus, alignment deformity, or effusion noted. Question What is the next appropriate step in the management of this patient?

This patient's manifestations are most suggestive of osteosarcoma. It is typically seen in patients in their second or third decade, occurs more frequently in men than in women, and is found in the metaphyseal areas of long bones, with 50% of lesions seen about the knee joint. The distal femur is the most common site, followed by the proximal tibia, and then the proximal humerus. The only blood tests with prognostic significance are lactic dehydrogenase (LDH) and alkaline phosphatase (ALP). Patients with an elevated ALP at diagnosis are more likely to have pulmonary metastases. In patients without metastases, those with an elevated LDH are less likely to do well than are those with a normal LDH. Other laboratory tests include a complete blood cell (CBC), including a differential and platelet level, electrolytes, and liver and renal function tests. The practitioner should also obtain plain films of the suspected lesions in 2 views. Elevation of the periosteum may appear as the characteristic Codman triangle; in approximately 60% of cases, extension of the tumor through the periosteum may result in a so-called sunburst appearance.

You are evaluating a 34-year-old African American man for a 5-week history of increasing right groin pain. He denies any injury or history of similar pain. The pain is worse with movement and has progressed to the point that the patient has severe pain with weight bearing. He denies fever, chills, urinary symptoms, or any other complaint at this time. He has a past medical history of sickle cell disease and hypothyroidism. Physical exam reveals tenderness upon palpation of the groin with increased pain on both active and passive range of motion of the hip. Homan's sign is negative. Distal pulses and sensation are intact and normal. Question What diagnostics study should be ordered at this time?

This patient's presentation is concerning for avascular necrosis of the right hip. The most appropriate diagnostic test to order at this time is an MRI of the hip with and without contrast. Avascular necrosis, also known as osteonecrosis, is a focal infarct of the bone due to an interruption of blood supply.1,2 Avascular necrosis (AVN) most commonly occurs in the hips, jaw, and scaphoid bone of the hand.2 Risk factors for AVN include trauma, coagulopathy, alcoholism, chronic corticosteroid use, sickle cell disease, and auto-immune diseases such as RA and SLE.

A 65-year-old woman presents to the office with decreased hearing, and pain over her sternum, pelvis, and her right tibial tubercle. On x-ray, the involved bones are noted to be expanded and denser than normal. Her serum calcium and phosphorus levels are normal, but serum alkaline phosphatase level is markedly elevated. Which of the following would be the appropriate initial treatment for this patient?

This patient's signs and symptoms are consistent with Paget disease of bone. Biphosphates have become the treatment of choice for this disease. Tiludronate, taken orally for 3 months, is very effective in treatment of this disease

A 76-year-old Caucasian woman presents to the emergency department after a fall. Upon further questioning, she states she has a history of rheumatoid arthritis that has been treated with steroids. Her doctor tells her that she has broken her hip. X-ray shows demineralization. Her doctor is concerned and runs some additional tests. A bone scan is done and shows reduced bone density. Her BMI is 21 kg/m2.

This woman has a history suggestive of osteoporosis. She developed a pathologic hip fracture secondary to the osteoporosis. Both the glucocorticoids she is on and the rheumatoid arthritis could have also contributed to her low bone mass. With osteoporosis there will be normal serum levels of calcium, phosphorous, and alkaline phosphatase. The chemical composition of the bone is normal but the density is reduced. Osteoporosis is asymptomatic until a fracture occurs.

"Toeing in"

Toeing in" in children before the age of 2 is typically due to tibial torsion; however, any "toeing in" after the age of 2 to 3, is usually due to femoral anteversion. The femur has more internal rotation that results in the presentation.

A 64-year-old female who has a history of injectable drug use presents with blood work that reveals leukocytosis with a left shift, and there is suspicion of osteomyelitis based on the patient's prior history. Based on this history, what bone would be most affected by hematogenous osteomyelitis in adults?

Unlike children, the long bones are rarely affected in adults with the vertebrae being the most likely location for the bone infection to occur. Lumbar vertebrae are most often affected, followed by thoracic and cervical vertebrae. Hematogenous osteomyelitis accounts for about 20% of all cases of osteomyelitis in adults. It is more common in males and the prevalence is higher amongst those who are IV drug abusers, patients being treated with dialysis or who have sickle cell disease

A 88-year-old female is found lying on the ground in a supine position after sustaining a fall in her house from tripping on the rug.. Her leg is shortened, abducted and externally rotated, and she is complaining of pain to the right leg with radiation to the knee. Based on the history of the patient, what type of injury has this patient sustained?

When a fracture occurs in the femoral neck that includes displacement, the leg appears shortened, abducted and externally rotated. When the leg appears shortened, flexed, adducted, and internally rotated, a posterior hip dislocation should be suspected. A leg that is flexed, abducted and externally rotated suggests an anterior hip dislocation. Posterior hip dislocations are far more common than anterior dislocations,

In a boxer's fracture, the presence of how much angulation of the fifth metacarpal neck would require referral for reduction as opposed to treatment with a simple ulnar gutter splint?

When there is greater than 40 degrees of angulation or an extensor lag (the patient cannot fully extend the affected finger) then a referral is required. Lesser degrees of angulation without an extensor lag can generally be handled conservatively with an ulnar gutter.

A 16-year-old boy presents following the striking of a wooden door with a closed fist an hour ago when he was angry at his mother. He is neurovascularly intact, and the skin is closed. There is an obvious deformity with a loss of small finger metacarpal knuckle. Radiographs reveal an oblique mid-shaft fracture of the 5th metacarpal with a palmar angulation of 45 degrees. Question What physical exam technique must be performed to check for rotation of the fracture?

With a fractured 5th metacarpal, there is an increased incidence of malrotation of the distal digit due to the potential loss of metacarpal height and lack of ligamentous support of the metacarpal head by the intermetacarpal ligaments. Therefore, rotation must be checked closely to ensure proper position of the distal fracture segment in relation to the proximal fracture fragment for functional healing to take place. This is best accomplished by asking the patient to make a partial fist (gently) and watching for finger malposition (normally all the digits of a closed fist point towards a single spot on the scaphoid) or "cross-over". Be sure to compare the injured hand to the uninjured one as some "normal cross-over" can be seen with the 5th metacarpal.

What is the first type of motion that is lost as a person develops progressively worsening osteoarthritis of the hip joint?

The earliest sign of the development of osteoarthritis of the hip is often the loss of internal rotation. As the condition worsens, muscular contractures may develop which hold the affected limb in a flexed and externally rotated position, which has great consequence on the individual's gait and functioning level. Typically a person experiencing this type of osteoarthritis will eventually develop an antalgic gait where the time spent bearing weight on the affected limb is brief due to pain. Also, the gluteus medius (which is a hip abductor and helps stabilize the pelvis) may become weakened as the condition worsens, resulting in an abductor lurch as the trunk of the body sways out over the affected limb when attempting to walk.

Which of the following is a type of primary malignant tumor of the bone?

Osteosarcoma is a type of primary bone cancer. Osteosarcomas are more common in the pediatric population, accounting for 5% of all childhood cancers and 56% of all cancers of the bone in patients less than 20 years old. Osteosarcomas have a bimodal occurrence rate with the highest prevalence between ages 11-13 and over the age of 65

A 32-year-old man is working on his farm when a wild horse suddenly jerks the bridle, forcing the man's right arm over his head and backwards. There is a "clunking" sound, and the man is in immediate pain and unable to move his arm. There is a deformity of the shoulder with a depressed area (dimple) noted in the anterior shoulder. Question What nerve must be carefully assessed in this patient before and after it is fixed because of the possible injury to it?

With a glenohumeral dislocation, as in this man, a risk of neurapraxia of the axillary nerve is possible. Neurovascular function, particularly that of the axillary nerve, must be carefully assessed before and after reduction of the dislocation. Anterior-inferior displacement of the humeral head into the quadrangular space has the potential of injury to the axillary nerve. The musculocutaneous nerve supplies the coracobrachialis muscle, leaves the axilla by piercing that muscle, and is not exposed to the force of dislocation. The median nerve arises from the lateral and medial cords of the brachial plexus, passes downward on the lateral side of the axillary artery, and also is not exposed to the force of dislocation. The radial nerve, the longest branch of the brachial plexus, lies behind the axillary artery and also is out of harm's way. The ulnar nerve arises from the medial cord of the brachial plexus, descends in the interval between the axillary artery and vein, and is also not subject to the force of dislocation.

A 12-year-old boy presents with left shoulder pain and deformity after a fall from his skateboard while attempting a half-pipe maneuver an hour ago. He heard a "cracking" sound and was unable to continue skateboarding. He has pain with movement of the arm in any direction. On exam, you note tenderness over the superior central shoulder with a palpable deformity and "tenting" of the skin. What is his most likely diagnosis?

With a superior central deformity and tenderness, along with tenting of the skin, the most likely diagnosis is clavicle fracture. AC separation would have deformity laterally on the shoulder. Glenohumeral dislocation would have shoulder contour deformity. Rotator cuff tear does not have acute deformity. Frozen shoulder is a slow, progressive chronic problem and not an acute injury.

A 39-year-old woman presents after getting hurt at her karate class the night before. She had been doing kick-jumps and landed on the outside of her left plantar-flexed and inverted foot with all of her weight. There was a popping sound, and sharp pain set in immediately. The instructor had ice put on the ankle and elevated it right away. Before leaving the facility, a compression wrap was put on. Treatment was sought about 14 hours later because of pain and swelling. On examination, swelling in the area in front of the lateral ankle is visible. The anterior drawer test shows a difference of 5mm between injured and non-injured ankle. The talar tilt test reveals no instability, and weight-bearing ability is only slightly restricted. Question What ligament is most likely affected?

anterior talofibular ligament

Level 1 evidence has shown vitamin C reduces the incidence of reflex sympathetic dystrophy (RSD) or complex regional pain syndrome type I (CRPS) in patients with which of the following? Review Topic QID:3356 1 Tarsal tunnel syndrome 0% (9/2523) 2 Distal radius fractures 92% (2315/2523) 3 Carpal tunnel syndrome 4% (91/2523) 4 Cervical radiculopathy from herniated nucleus pulposis 1% (25/2523) 5 Ankle fractures

distal radial fractures

Ankle strains

involve the stretching or tearing at the junction where the muscle meets the tendon (sprains have to do with stretching or tearing of the ligament). Acute strains will present with pain, muscle spasms, loss of strength, and potentially limited range of motion. Strains take place with the muscle is stretched and suddenly contracts, as is the case in running or jumping

Takayasu's arteritis

is a granulomatous vasculitis of medium sized and larger arteries. Takayasu first described it in 1908 as a clinical syndrome characterized by ocular disturbances and marked weakening of the pulses in the upper extremities (Pulseless disease). This is related to the fibrous thickening of the aortic arch with narrowing, or virtual obliteration, of the origins or more distal portions of the great vessels arising in the arch. Microscopically, there is adventitial mononuclear infiltrate with perivascular cuffing of the vasa vasorum (initially). Later, there is intense mononuclear inflammation in the media, in some cases accompanied by granulomatous changes replete with giant cells and patchy necrosis of the media. In addition, when it heals, the inflammation is replaced by marked collagenous fibrosis involving all the layers of the vessel wall accompanied by lymphocytic infiltration.

Streaking erythema

is a sign of lymphangitis, which is inflammation or infection that has spread to the lymphatic system.

Fibromyalgia

is a soft tissue, non-articular pain disorder characterized by chronic, generalized musculoskeletal aches, pains, and stiffness that occur primarily in muscles and their attachments. It is associated with specific sites of exaggerated tenderness. Palpation of the upper edge of the trapezii; neck muscle insertion at the occiput, deltoids, infraspinatus, and second costochondral junction; inferior to the lateral (elbow) epicondyles, trochanters, and lower lumbar area; medial fat pads of the knees; and the medial and lateral insertions of the Achilles tendons reveal 12 or more tender points. The first treatment is patient education. The client will feel better just knowing they have a diagnosable syndrome that can be managed. The use of antidepressant drugs can help. Drugs like amitriptyline, fluoxetine, chlorpromazine, or cyclobenzaprine are used. Exercise is another important aspect of treatment

Kawasaki disease

is an acute arteritis involving large, medium sized, and small arteries (often the coronary arteries) and is associated with mucocutaneous lymph node syndrome. The mucocutaneous syndrome is characterized by fever, conjunctival and oral erythema, edema of the hands, feet, erythema of the palms and soles, a skin rash (often with desquamation), and enlargement of the cervical lymph nodes. It is usually self-limited. Approximately 20% develop cardiovascular sequelae, with the range of severity from asymptomatic vasculitis of the coronary arteries, coronary artery ectasia, or aneurysm to giant coronary artery aneurysm (7 to 8 mm) with rupture or thrombosis, myocardial infarction, or sudden death. Acute fatalities occur in 1% of the cases due to coronary artery thrombosis or rupture of coronary artery aneurysm. Vasculitis resembles polyarteritis nodosa with necrosis and pronounced inflammation, affecting the entire thickness of the vessel wall.

Rheumatic fever

is an inflammatory condition that is precipitated by group A streptococcal infection. It is more common in children The symptoms include joint pain, carditis, fever, subcutaneous nodules, abdominal pain, nosebleeds, and chorea

Reiter syndrome, also called reactive arthritis (ReA),

is not consistent with other spondyloarthropathies. It is a triad of arthritis, nongonococcal urethritis, and conjunctivitis. The syndrome characteristically is an acute, asymmetric arthritis of the lower extremities that may involve the sub talus or present a dactylitis (sausage digit). It may be isolated to joints or include systemic illness, characterized by fatigue, fever, and weight loss.

Lymphadenopathy

is swelling of a lymph node secondary to infection, autoimmune disease, or malignancy.

Polyarteritis nodosa

is systemic vasculitis characterized by necrotizing inflammation of the small or medium sized vessels, typically involving renal arteries and visceral vessels but sparing the pulmonary circulation. There is neither glomerulonephritis nor vasculitis of the arterioles, capillaries, and venules. It particularly involves the branching points, also known as the points of bifurcation. The involvement is segmental and may involve only a portion of the circumference. It causes segmental erosion with weakening of the arterial wall with aneurysmal dilatation or localized rupture. Initially, there is transmural inflammation of the vessel consisting of neutrophils, eosinophils, and mononuclear infiltrate with fibrinoid necrosis. Later, the inflammation is replaced by collagenous fibrosis. Within the same vessel, various stages of inflammation may be seen.

Inversion ankle sprains

will present with localized pain and swelling over the lateral aspect of the ankle only. The patient would have difficulty weight bearing and would still attempt to walk, although the patient would most likely limp.

A 38-year-old man presents with pain and the inability to extend his middle finger DIP joint following a sudden jamming-type injury when attempting to catch a football 3 hours ago. His radiographs are normal. Based on the patient history, what is the most likely diagnosis?

A mallet finger deformity is due to the rupture of the extensor tendon at the base of the dorsal distal phalanx of any digit of the hand. Patients will report pain dorsally at the DIP joint with the inability to actively extend the DIP joint.

e fracture of a patient's lower humerus results in the inability to extend the wrist. What nerve was damaged in the injury?

Damage to the median and ulnar nerves would limit the ability to flex the wrist and fingers. Damage to the musculocutaneous nerve would limit the ability to flex the forearm. Therefore, because the radial nerve innervates the extensor (posterior) forearm muscles, radial nerve must be the correct answer.

A 48-year-old woman presents with pain on the lateral aspect of her right dominant wrist. It began 3 days ago after playing video games with her son. She does not remember any specific injury, but she had to use a pencil to figure her taxes. She has pain now trying to hold a pencil to write. What physical findings would you most likely find?

Flexion of the thumb into the palm along with ulnar deviation of the wrist (Finkelstein test) will increase the pain of the lateral wrist due to stretching of the tendons of the 1stdorsal compartment (the extensor pollicus brevis and the abductor pollicus longus), which are swollen and tender.

Pediatric Wrist/Elbow Xrays

In pediatric patients, there are multiple growth centers in the elbow. Therefore, it is recommended to take the contralateral elbow to compare the anatomic positions of the growth centers along with the injured elbow AP and laterals. The wrist AP and lateral should also be taken because of the possible injury to the wrist.

CERVICAL RADICULOPATHY

Pain on the symptomatic side on putting the ipsilateral ear to the shoulder suggests radiculopathy, but increased pain on leaning or turning away from the symptomatic side suggests a myofascial origin Light digital compression of the jugular veins until the face is flushed and the patient is uncomfortable will sometimes elicit radicular symptoms: unilateral shoulder, arm, pectoral or scapular pain, or radiating paresthesias into the arm or hand (Naffziger's sign).

A 55-year-old woman presents with weakness, bone pain, and lethargy. Serological tests reveal the presence of Russell bodies. The accumulation of what intracellular material is most likely responsible for the presence of Russell bodies?

Russell bodies are the accumulation of immunoglobulin in plasma cells; immunoglobulins are proteins.

adhesive capsulitis,

Suspect a frozen shoulder, also known as adhesive capsulitis, if shoulder pain comes on gradually, limiting the motion at both the glenohumeral and scapulothoracic joint. Often there will be limitation of abduction, external rotation, and forward flexion. The hallmark of frozen shoulder is a spontaneous onset of pain that worsens gradually and insidiously, followed by loss of range of motion. The pain may radiate, but pain below the elbow is not common. Pain at night is also a common symptom.

An 80-year-old man presents to your office with low back pain. Which of the following is the least likely etiology? Prostatitis Abdominal aortic aneurysm Osteoarthritis Osteopenia

The correct answer is D. Prostatitis, abdominal aortic aneurysm, and osteoarthritis are all possible reasons for low back pain in an 80-year-old patient. Osteopenia (low bone mass) is typically asymptomatic.

palpable calf cord

can be appreciated in patients who present with a DVT or varicose veins

Which of the following joints has the lowest occurrence rate of osteoarthritis?

Because the elbow is not a weight bearing bone, the rate of osteoarthritis in the elbow is considerably less that what is found in locations like the hips, knees and spine. The hands have one of the highest rates of occurrence of osteoarthritis, likely due to their near constant use and propensity for minor (or major) injury. When elbow arthritis does develop it is often post-traumatic osteoarthritis

A 35-year-old male presents with pain and decreased range of motion after sustaining a fall in which the patient tried to grab onto a bar which pulled his entire arm in the process. Given this clinical scenario, at what cervical motor neuron level would the biceps reflex be testing?

C5 is the primary motor neuron being tested in a biceps reflex. C6 contributes to the brachioradialis reflex primarily, but does have a small role in the biceps reflex. The C7 motor neuron is primarily involved in the triceps reflex. C4 and C8 do not contribute to any primary reflexes.

A 65-year-old man with uncontrolled diabetes presents for evaluation of a foot injury. He ran into his dresser 3 days ago, and an ulceration has developed at the site of the injury. He admits to minimal localized pain, and he denies any associated fever or chills. On examination, the ulceration measures 2.5 cm in diameter, and a sterile probe can be easily advanced to the underlying bone.

MRI has been identified as the most sensitive imaging modality for the diagnosis of osteomyelitis, and it is particularly helpful in identifying soft tissue involvement.

Which peripheral nerve is involved in the most common compression neuropathy in the upper extremity?

Median

jerk test

The jerk test evaluates posterior capsule instability. The patient's arm is placed in 90° of flexion and maximum internal rotation with the elbow flexed at 90°. The arm is then adducted across the body in the horizontal plane while pushing the humerus in a posterior direction. The test is positive if a posterior subluxation or dislocation occurs

A 20-year-old football player presents with a complaint of knee pain after "being hit on the outside of his knee when carrying the football." On physical exam, he has localized tenderness over the medial joint line along with laxity and pain on valgus stress of the knee. Which of the following is the most likely diagnosis? Lateral collateral ligament tear Patellar dislocation Chondromalacia patella Medial collateral ligament injury

The correct answer is D. Trauma to the outside of the knee often causes medial collateral ligament (MCL) injuries. They typically present as described in the question. Lateral collateral ligament (LCL) injuries are lateral injuries. Patellar dislocations do not cause laxity on ligamentous testing of the knees.

Giant cell arteritis

or temporal arteritis, is the most common of the vasculitides. It is an acute and chronic granulomatous inflammation of the medium and small arteries. It affects mainly the temporal, vertebral, and ophthalmic arteries. The short segments of one or more arteries develop nodular thickening with the reduction of the lumen, which may become thrombosed. Histologically, there is granulomatous inflammation of the inner half of the media, centered on the internal elastic membrane. It is marked by mononuclear infiltrate, multinucleated giant cells of both foreign body and Langhans-type, and fragmentation of the internal elastic lamina. Sometimes the granulomas may be absent, and there is only nonspecific polyarteritis without giant cells. The healed stage of both will reveal only collagenous thickening of the vessel wall.

OSTEOARTHRITIS

Morning stiffness lasting <30 minutes, stiffness that improves with activity, Bouchard's nodes (PIP joints), Heberden's nodes (DIP joints), and crepitus. Among people 55 years and older, women are affected more often than men. On examination, patients with OA of the hip and knee may present with an antalgic gait; patients are limping to avoid pain on the affected hip and/or knee. Examine bilateral quadriceps muscles for signs of weakness. Internal and external hip rotation may be reduced The use of NSAIDs should be avoided in older adults with a calculated creatinine clearance <35 mL/min.

How often should she be recommended to follow-up with a DXA bone density scan?

Recommendations for patients diagnosed with osteoporosis without a fracture is every 2-3 years (C). This guideline is approved based on age, risk factors, or previous fractures. A post-menopausal woman at age 64 without a presenting fracture can be followed every 2-3 years with bone density imaging

A 65-year-old woman presents to the office with decreased hearing, and pain over her sternum, pelvis, and her right tibial tubercle. On x-ray, the involved bones are noted to be expanded and denser than normal. Her serum calcium and phosphorus levels are normal, but serum alkaline phosphatase level is markedly elevated. Which of the following would be the appropriate initial treatment for this patient?

This patient's signs and symptoms are consistent with Paget disease of bone. Biphosphates have become the treatment of choice for this disease. Tiludronate, taken orally for 3 months, is very effective in treatment of this disease.

What is seen during the early stages of osteoarthritis?

Thickening of articular cartilage

A 67-year-old man presents with pain and stiffness in his shoulders and hips lasting for several weeks with no history of trauma. He also has complaints of headache, throat pain, and jaw claudication. It is imperative to diagnose this patient promptly in order to prevent which of the following complications?

The most urgent need for diagnosis of a patient with symptoms of polymyalgia rheumatica (PMR) and giant cell arteritis is to prevent blindness caused by ischemic optic neuropathy as a result of occlusive arteritis of the ophthalmic artery.

vitamin D deficiency

Vitamin D is activated in the kidneys. The first step in the synthesis of Vitamin D is the conversion of 7-dehydrocholesterol to cholecalciferol in the skin by ultraviolet light. Several metabolic conversions must take place in order to obtain the active hormone. This pro-hormone is then further activated in the liver by the enzyme 25-hydroxylase to form 25-hydroxycholecalciferol. This is the form of vitamin D found in the circulation. The 25-hydroxy-cholecalciferol is further converted to 1, 25 dihydroxycholecalciferol in the kidney by the enzyme 1-alpha hydroxylase. This is the active form of the vitamin. The softening of the bones in an adult is called osteomalacia. It is also known as adult rickets because it is caused by a lack of vitamin D and calcium in the diet. The bone structure remains normal, but the softening occurs due to defects in the normal calcification process. Vitamin D deficiency is seen in patients with inadequate sunlight exposure, such as elderly nursing home patients who also ingest inadequate amounts of vitamin D. Malabsorption of the vitamin can also cause a deficiency. Severe liver or kidney disease as well as genetic defects in the vitamin D receptor are other causes of this condition.

A 27-year-old female presents to the emergency department after a motor vehicle accident. Imaging of the left lower extremity shows the following fracture pattern: How would this fracture be described? A No displacement B Complete dorsal displacement C Fifty percent dorsal displacement D Displacement with shortening

This Fracture is displaced, pertaining to any deviation from anatomical position or alignment to the extent to which the fracture fragments are nonconcentric or offset from each other. This fracture is only 50 percent displaced without angulation or shortening. The magnitude of displacement is expressed in either terms of measurement (i.e. incomplete) or percentage (i.e. 50%) ) of the width of the bone. The direction of displacement is based on the position of the distal fragment relative to the proximal fragment. Separation is the distance two fragments have been pulled apart. Shortening (D) is the amount the bone's length has been reduced, which is not seen in this fracture. Angulation is the degree of "bending" that makes the fragments unparalleled. This would not be described as "no displacement" (A) or "complete displacement" (B).

Case A 54-year-old man presents with a 2-month history of left, non-dominant shoulder pain. There is no history of trauma, but the pain began about a week after shoveling wet, heavy snow from his 100-foot driveway. At first, the pain seemed to come and go from day to day, but has gotten progressively worse and more constant. Pain is worse with overhead use, and he cannot sleep on his left side; he wakes up if he rolls over onto his left shoulder. On exam, he is tender to palpation over the left humeral greater tuberosity. He has pain with Neer and Hawkins tests, but has a negative drop arm test. You prescribe physical therapy and non-steroidal anti-inflammatories, and you also advise taking a break from using the arm for heavy work.. Question At what point would you recommend a steroid injection?

The correct response is only after failure of a home therapy program of at least 6 weeks. When a home therapy program performed 3 - 4 times a day for 6 weeks fails to improve the symptoms of impingement syndrome, a subacromial injection of steroid could be considered. Posterior capsule stretching exercises should be performed at home prior to any steroid injection. Some patients require the supervision of a physical therapist to assure adequate stretching, but they still require at least 6 weeks of therapy prior to an injection. Steroid injections should not be repeated if the 1st one does not improve the symptoms for at least 4 - 6 weeks. Repeated steroid injections can weaken the tendons of the rotator cuff and potentially cause a rupture of the tendon.

Upon testing a patient for function of the hip flexors, which muscle is considered the primary muscle responsible for most flexion?

The iliopsoas muscle is the primary hip flexor muscle. It originates at T12 and L1-5 vertebrae and intervertebral disks as well as the iliac fossa of the pelvis and connects to the femur at the lesser trochanter. The gracilis muscle is considered a secondary hip adductor. Rectus femoris does help with hip flexion, but in a secondary role to the iliopsoas. Rectus femoris is also involved in knee extension. Sartorius is also involved in hip flexion, but in a secondary role. The

systemic lupus erythematosus.

The patient's presentation is characteristic of systemic lupus erythematosus. It is a non-organ specific, autoimmune disease. It is 9 times more common in women, especially during pregnancy, and almost 10 times more common in West Indian blacks. It is a disease of relapses and remissions. Musculoskeletal system involvement occurs in the vast majority of cases, commonly as arthralgia, myalgia, and myositis. Photosensitive butterfly rashes, scarring alopecia, livedo reticularis, oral ulceration, and conjunctivitis are the common cutaneous manifestations. Other common features include: fever, splenomegaly, psychosis, fits, cranial nerve involvement, and meningitis. Renal symptoms include proteinuria, edema, hypertension, and renal failure. Pleurisy and pleural effusion as well as pericardial effusions may occur together with endocarditis and cutaneous vasculitis. 80% have positive antinuclear antibody, but high titer of anti-double stranded DNA is almost exclusive to SLE. 40% have a rheumatoid factor. antibodies to double-stranded DNA

A 52-year-old woman, with a body mass index of 35, presents for evaluation of back pain. The pain becomes worse at night and is accompanied by stiffness. For the past week, she has been experiencing night sweats and fever. She relates that she had a positive PPD test a year ago and did not follow up as directed. An X-ray of her lumbar spine reveals osteopenia and cortical breakdown of vertebral bodies L4 and L5. Question What is the most likely diagnosis?

The clinical picture is suggestive of tuberculosis of the spine (Pott disease).

Upon testing a patient for function of the hip abductors, which muscle is considered the primary muscle responsible for most abduction?

The primary mover in the motion of hip abduction is the gluteus medius muscle. Gluteus minimus does play a supporting role in that motion. Biceps femoris is one of the three hamstring muscles and contributes to the motions of knee flexion (primary muscle) and hip extension (secondary muscle). Gluteus maximus is the primary mover for hip extension

What types of connective tissue are injured in a sprain?

A sprain involves injury to those tissues that give support to joints - ligaments and joint capsules. Injury to muscles, tendons, and fascia would all be classified as a strain. Injuries to bone would be classified as a fracture.

risk factors for OA include:

Advanced age Obesity (due to increased weight-bearing stress on the joints). The knee joints are most susceptible. Menopause Trauma Excessive use Infection Certain hemoglobinopathies, previous rheumatoid arthritis, acromegaly, etc. Before the age of 55, OA affects women and men equally; however, after 55, it has been found to be more common in women.

A parent brings her child into the office with the concern of the way he stands. To the parent the child looks like they are "bowlegged." If the child is indeed bowlegged, what would be the best description of this orthopedic abnormality?

Angulation of an extremity at a joint with the more distal part angled toward the midline.

A 52-year-old male has a 200-pound file cabinet fall on his right leg. He comes to the emergency department complaining of pain and swelling to the right leg. He is also complaining of parasthesias to the leg also. Based on these findings on history, what part of the leg would be the most likely site of compartment syndrome?

Anterior compartment of the leg

Out of all cervical vertebrae, which two are responsible for the greatest amount of rotation? A C1 & C2 B C2 & C3 C C3 & C4 D C4 & C5 E C5 & C6

Approximately 50% of cervical rotation takes place between the C1 (atlas) and C2 (axis) vertebrae. These first two cervical vertebrae have a different shape from the other cervical vertebrae that allow for this greater range of motion. The remaining 50 % of cervical rotation is split fairly evenly between the remaining vertebrae. Approximately 50 % of flexion and extension occurs between the occiput at the base of the skull and C1 with the remaining 50% distributed fairly evenly between the remaining vertebrae with a slightly higher percentage occurring at the C5 & C6 level

The diagnosis of systemic lupus erythematosus (SLE) is supported by a positive initial antibody screen; however, this test is not specific. Which of the following tests is most specific in the diagnostic evaluation of SLE?

Autoantibody production is the primary immunological abnormality seen in patients with systemic lupus erythematosus (SLE); the antinuclear antibody (ANA) is most characteristic of SLE and seen in 95% of patients with SLE but is not specific for the diagnosis of SLE. A positive ANA can also be found in patients with lupoid hepatitis, scleroderma, rheumatoid arthritis, Sjögren disease, dermatomyositis, and polyarteritis. ANA testing should be employed as the initial screening test in a patient suspected of having SLE. A negative total ANA test is strong evidence against the diagnosis of SLE, whereas a positive test is not confirmatory of the diagnosis. The most specific antibody tests for SLE are antibodies to double-stranded DNA (anti-dsDNAs) and anti-Smith (anti-SM).

A 49-year-old man, who is well known to your practice, presents with pain in his left lower extremity. Upon further questioning, the patient describes doing construction work 3 months prior during which he jumped onto his feet from an elevated height of about 5 ft. Since this episode, he has noted increasing issues of left-sided hip and knee pain. He describes the pain as radiating into the left groin and front middle thigh area. The pain is relieved with sitting, and it is aggravated by walking and climbing up stairs. The patient denies paresthesias, numbness, bowel/bladder dysfunction, fever, night sweats, and chills. Radiographs performed reveal marked irregularity of the left femoral head with sclerosis, subchondral lucency, and the presence of mild collapse.

Avascular necrosis,

A 19-year-old boy presents with pain and deformity of his right dominant shoulder after a sudden jerking movement to the same from a wrestling competitor approximately 1 hour ago. He states he felt a clunking sensation when it happened. He was unable to continue wrestling and has pain with movement of the right shoulder. After returning from radiology, you review the films and diagnose a glenohumeral dislocation. After reducing the dislocation, what part of the physical exam MUST be done again?

Axillary nerve palsy may develop with reduction of a glenohumeral dislocation. It is imperative that both pre- and post-reduction neurological exams be performed.

A 30-year-old male with a known seizure disorder presents to the emergency department with confusion in an obvious postictal state along with significant left shoulder pain. He is unable to move his left shoulder. The radiograph is shown below. Which of the following is the most likely diagnosis? Anterior shoulder dislocation Posterior shoulder dislocation Humeral shaft fracture Scapular or clavicular fracture

B. Posterior shoulder dislocations are uncommon, but classic seizure injuries. Anterior dislocations are much more common, but not commonly seen after seizures. Scapular fractures are rare, and the diagnosis of clavicular fractures is much less challenging. The clinician should note that frequently, there is a delay in the diagnosis of posterior shoulder dislocations.

An 80-year-old man presents with a complaint of pain in his upper arm. He states that he heard a "pop" in his upper arm while bowling last night and immediately afterward felt pain and was unable to use the arm to lift a bowling ball. On physical exam, he has ecchymosis of the upper arm and a visible bulge about halfway down his upper arm. Which of the following is the most likely diagnosis? Anterior shoulder dislocation Biceps tendon tear Elbow fracture Acromioclavicular joint sprain

B.The biceps tendon frequently tears in older patients, especially after a resisted contraction of the biceps itself. Over 90% occur proximally, almost exclusively involving the long head. Often the ecchymosis will be quite impressive and be present over much of the upper arm. A lump may be present in the mid-humerus area as this area's muscle generally contracts following rupture. It is unusual to have ecchymosis in the arm with the other injuries. A fall was not involved, which would be a more common mechanism with the other injuries.

What nerve is most commonly injured in a mid- or distal humeral shaft fracture?

Because of the radial nerves proximity to the humerus, mid and distal shaft fractures with significant displacement can cause a radial nerve injury. Median and ulnar injuries are more commonly associated with forearm injuries. Axillary nerve injuries are most common in anterior shoulder dislocations and peroneal nerve damage occurs as a result of lower leg insult. (

What is the most common joint dislocation in children?

In children the elbow is the most commonly dislocated joint and it is the third most common joint dislocation for adults. The shoulder and finger are dislocated more frequently than the elbow in adults.

Upon testing a patient for function of the hip extensors, which muscle is considered the primary muscle responsible for most extension?

Gluteus maximus

A fracture involving the medial epicondyle will most likely cause damage to which nerve?

Ulnar

ulnar nerve

supplies the 3 muscles of the hypothenar eminence, the palmaris brevis, the 3rd and 4th lumbrical muscles, all the interossei muscles, and the adductor pollicis muscle

Parents bring their 4-year-old daughter in because of knock-knee. She is otherwise healthy, and her height is in the 50th percentile for age. On examination, she has about 10 degrees of valgus. What should you recommend?

Children are normally bowlegged (genu varum) at birth. By 12 to 18 months of age, the legs become straight. Then children develop knock-knee (genu valgum), which is maximal by about age 3-4 years. This averages 10-15 degrees at this age. The legs then spontaneously "straighten" to 5-10 degrees of valgum, which is the average for adults. This child is within the normal range, and the parents should be reassured.

The patient is a 40-year-old woman who is G5 P4. On ultrasound at 24 weeks, the fetus is diagnosed as small for gestational age (SGA). Normal heart, stomach, situs, and cord vessels are identified. The fetus is identified as having a protruding myelomeningocele during the sonographic examination. During the examination, the sonographer noted reduced movement in the lower limbs. This male fetus has associated malformations that are common with this malformation. What is associated with a lumbar myelomeningocele?

Clubfoot is often seen with a lumbar myelomeningocele, possibly due to the lack of nerve stimulation to the muscles. The basic deformity is a subluxation at the talocalcaneonavicular joint. Several factors, including family history and intrauterine crowding, have been blamed as the causative factors. It is associated with meningomyelocele, arthrogryposis multiplex congenita, and tibial hemimelia.

An elderly woman sustained a fracture of the proximal humerus 4 days ago while she was out of town visiting her daughter. She was seen in an Urgent Care clinic there and was told to follow up with her doctor when she got home. They placed her arm in a shoulder immobilizer and told her to leave it on. She was and is neurovascularly intact distal to her fracture. She comes to your office now for further care. What physical exam finding should be expected after 3 or 4 days?

Common sites that are affected include the proximal or distal femoral head or even the ankle, shoulder, or elbow. Causes include, but are not limited to corticosteroid use, alcoholism, trauma, systemic lupus erythematosus, pancreatitis, gout, or even sickle cell disease. Radiographic findings will include mild density changes early on and sclerosis, while a more progressed disease state will lead to the pathognomonic crescent sign, which is subchondral collapse. Later disease will show joint-space narrowing and degenerative changes in the acetabulum. The age of this patient would make the treating healthcare provider highly pursue conservative efforts to help preserve the affected joint. Unfortunately, with progressive process of this disease, most likely the patient in the above scenario will need to have a total hip replacement. The initial treatment plan usually includes avoidance of weight bearing of the affected joint for several weeks. Vascularized or nonvascularized bone grafting is sometimes used to help preserve the femoral head but only delays the ultimate clinical intervention of a total hip arthroplasty.

Avascular necrosis

Common sites that are affected include the proximal or distal femoral head or even the ankle, shoulder, or elbow. Causes include, but are not limited to corticosteroid use, alcoholism, trauma, systemic lupus erythematosus, pancreatitis, gout, or even sickle cell disease. Radiographic findings will include mild density changes early on and sclerosis, while a more progressed disease state will lead to the pathognomonic crescent sign, which is subchondral collapse. Later disease will show joint-space narrowing and degenerative changes in the acetabulum. The age of this patient would make the treating healthcare provider highly pursue conservative efforts to help preserve the affected joint. Unfortunately, with progressive process of this disease, most likely the patient in the above scenario will need to have a total hip replacement. The initial treatment plan usually includes avoidance of weight bearing of the affected joint for several weeks. Vascularized or nonvascularized bone grafting is sometimes used to help preserve the femoral head but only delays the ultimate clinical intervention of a total hip arthroplasty.

A young mother brings her 5-week-old female infant for a routine check up. Pregnancy and delivery had been uneventful. The infant is doing fine, but the mother tells you that she has problems spreading the girl's left leg when she is changing her diaper. You suspect a dislocated hip. What should be looked for in order to determine whether or not the infant has a dislocated left hip?

Congenital dislocation of the hip is more common in female infants, infants with a positive family history, and breech in utero positioning. Gravity will cause the femur to drop toward the bed, which makes the affected femur appear shorter when the baby is held in a supine position. Hip X-rays are very difficult to interpret, especially at an early age. Therefore, they are not the method of choice for determining whether or not an infant has a dislocated hip. Hip ultrasounds are very accurate in early diagnosis. There would also be an increase in skin fold on the affected side. Since the major sign of hip dislocation or subluxation is the Ortolani's test, the inability to completely abduct the thigh to the surface of the examining table when the hip and knee are flexed will be present on the left side. This is not typically a painful test and should not produce discomfort in the patient. The Trendelenburg's sign is positive in persons with congenital hip dislocation and pelvis abnormality. When the person stands on the affected leg, the opposite (normal) gluteal fold falls. This test is almost impossible to do on an infant.

Anterior interosseous nerve

supplies to the flexor pollicis longus, the pronator quadratus, and the lateral half of the flexor digitorum profundus, as well as to the wrist, carpus, and distal radioulnar joints.

Polymyalgia Rheumatica

symptoms persist for more than 2 weeks and include morning stiffness lasting more than 2 hours of the proximal limb muscles, shoulder, and hip girdles.

The most commonly fractured long bone in both adults and children is which of the following

The tibia is the most commonly fractured long bone in the body for both adults and children. The fractures are often the result of sporting activities in the young and may occur from a simple fall in the elderly - especially those with osteoporosis. Motor vehicle accidents are another common cause of tibial fractures.

A 40- year-old female patient comes to the office because she notices that she is easily fatigued but cannot pinpoint a direct cause of her fatigue. She has experienced muscle tenderness to the shoulders and other large muscle groups, sensitivity to touch these areas, and has also felt depressed lately. Based on this history and the vague physical exam findings, what is the most likely diagnosis?

Fibromyalgia syndrome (FMS) primarily affects woman between the ages of 20 and 60 and is now the second most common condition seen in rheumatologists' offices behind only rheumatoid arthritis.

Approximately what percentage of patients with a solid primary tumor elsewhere will end up with metastatic disease of the vertebrae during the clinical course of their cancer? A 10% B 20% C 30% D 40% E 50%

Fifty percent of cancer patients will develop metastatic disease of the vertebrae at some point during the course of their illness

Approximately what percentage of patients with a solid primary tumor elsewhere will end up with metastatic disease of the vertebrae during the clinical course of their cancer?

Fifty percent of cancer patients will develop metastatic disease of the vertebrae at some point during the course of their illness. The highest percentages of cancers that lead to such spinal lesions are carcinomas of the breast, lung, prostate, colon, thyroid and kidney

A 7-year-old is diagnosed with an acute case of hematogenous osteomyelitis accompanied with fever and leukocytosis. Based on your knowledge of the disease, which bone is most likely to present with the infection?

Fortunately, hematogenous osteomyelitis is not common in children, but when it does occur it primarily is found in the long bones.

A 40-year-old accountant enters your office with an obvious limp. He complains of pain near the back of his ankle. He notes that he was running the bases yesterday during a softball game and heard a "pop" in the back of his ankle and felt pain in the same area. He could not run after the injury. On physical exam, he has ecchymosis from his heel up to his calf, a palpable defect above his calcaneus posteriorly, and is unable to plantar flex the foot against resistance. Which of the following is the most likely diagnosis? Achilles tendon rupture Ankle sprain Calcaneal bursitis Peroneal tendonitis

The correct answer is A. An ankle sprain can happen this way, but it will not cause the palpable defect of the Achilles. Peroneal tendonitis does not cause any degree of ecchymosis.

A 52-year-old overweight woman has had pain in her right hand for the past month. She is employed as a pastry chef and has trouble making a fist. On exam, she is tender over the radial styloid. You have her flex her thumb into her palm and move the wrist into ulnar deviation. This movement recreates her pain. What is the initial treatment for this condition?

Explanation DeQuervain's disease is a type of tenosynovitis. The initial treatment is conservative, involving NSAIDs and splinting. If conservative therapy fails, then steroid injections (up to 3) may be tried.

If the musculocutaneous nerve were to become damaged, what function would be lost?

Explanation The major muscles innervated by the musculocutaneous nerve are the brachialis and biceps brachii. Therefore, forearm flexion would be lost. Forearm extension is controlled by the triceps brachii which is innervated by the radial nerve. The wrist flexors are innervated mostly by the median and ulnar nerves. Wrist extensors are innervated by the radial nerve. The pronators (pronator quadratus and pronator teres) are innervated by the anterior interosseous and median nerves.

A 30-year-old female presents with a year history of chronic aching pain and stiffness throughout her entire body. She denies any trauma, new activities, or change in symptoms throughout the day. Her past medical history is significant for trouble sleeping at night and irritable bowel symptoms. On physical examination, the only finding is pain produced on palpation of the trapezius, medial knee, and lateral epicondyle of the elbow bilaterally. Her vitals are normal. All laboratory work including a thyroid panel, complete metabolic panel, erythrocyte sedimentation rate, and rheumatoid factor are within normal limits. What is the most likely diagnosis based on these findings?

Fibromyalgia (B) is a diagnosis of exclusion most frequent in women aged 20-50 years. The patient with fibromyalgia complains of chronic aching pain and stiffness, frequently involving the entire body but with prominence of pain around the neck, shoulders, low back, and hips. Fatigue, sleep disorders, subjective numbness, chronic headaches, and irritable bowel symptoms are common. Objective signs of inflammation are absent and laboratory studies are normal. Physical examination is normal except for "trigger points" of pain produced by palpation of various areas such as the trapezius, the medial fat pad of the knee, and the lateral epicondyle of the elbow. Polymyositis (A) produces weakness rather than pain. Polymyalgia rheumatica (C) produces shoulder and pelvic pain and is associated with an elevated ESR, occurring after age 50. Rheumatoid arthritis (D) and systemic lupus erythematosus (SLE), (E), present with objective physical findings or abnormalities on routine testing.

"crescent sign"

If the "crescent sign" is seen on plain radiography (x-ray), AVN is confirmed.

owing injury to a nerve at the wrist, the thumb is laterally rotated and adducted. The hand has a flattened appearance and is "ape-like." What nerve is damaged?

In the palm, the median nerve supplies the 3 muscles [(i) abductor pollicis brevis, which abducts the thumb at carpometacarpal joint and metacarpophalangeal joint; (ii) flexor pollicis brevis, which enables the thumb to form one claw in the pincer-like action used in the picking up of objects; and (iii) opponens pollicis, which enable the thumb to adduct at the carpometacarpal and at the metacarpophalangeal joint] of the thenar eminence. All 3 are supplied by the median nerve. Injuries to the nerve make the thumb laterally rotated and adducted and makes the hand have a flattened appearance.

A 66-year-old man presents with monoarticular arthritis. He has stage 3 chronic kidney disease. His affected joint is warm to the touch. You suspect gout. He is allergic to aspirin. What anti-inflammatory should you prescribe?

In this case, you are looking for an anti-inflammatory agent that will not cause a decline in renal function. Prednisone is recommended. Aspirin has anti-inflammatory properties, but it is contraindicated because the patient is allergic to it. Acetaminophen could be use for analgesia, but it is unlikely to decrease inflammation. Metformin is a medication used for the treatment of diabetes. Ibuprofen is an anti-inflammatory and anti-pyretic agent. Ibuprofen will likely cause a decline in renal function; it and other non-steroidal medications block prostaglandin-mediated vascular relaxation, effectively vasoconstricting and decreasing glomerular filtration rate and renal function. Approximately 30% of patients exposed to non-steroidal medications will have a decline in renal function. Patients with pre-existing kidney disease may be more susceptible to a decline in function, as they may be more dependent on the above-described prostaglandin-medicated vascular tone.

A 27-year-old woman fell onto her outstretched hand while playing tennis. She demonstrates exquisite tenderness with palpation of the anatomical snuffbox. Her radiograph is shown below. Which of the following is false regarding this patient? Save Image Enlarge Image Nonunion and osteonecrosis are common complications of scaphoid fractures. The bone scan shows decreased uptake in the area of the scaphoid. Radiographs should be taken again 10 to 12 days post injury. If anatomical snuffbox tenderness is present, treat it as a scaphoid fracture until proven otherwise.

The correct answer is B. The correct answer is the use of short half-life radiopharmaceutical agents to visualize bones. A bone scan would show increased, not decreased, uptake in an area of concern, such as a fracture, osteomyelitis, or metastases.

40. During a routine examination of an 8-week-old infant, you place the infant supine while supporting the head, back, and neck. You quickly lower the infant's body about 2 feet and notice abduction and extension of his arms, opening of his hands, and flexion of his legs in response to this maneuver. Which of the following infant reflexes does this best describe? Asymmetric tonic neck reflex Galant reflex Moro reflex Parachute reflex

Save Image Enlarge Image The correct answer is C. This correctly describes the Moro, or startle reflex, which a clinician can elicit in an infant from birth up to 4 months of age. The asymmetric tonic neck reflex usually lasts up to 2 months of age. To demonstrate the asymmetric tonic neck reflex, the examiner places the baby in a supine position and then turns the baby's head to one side. The arms and legs ipsilateral to the side the head is turned extend, whereas the arm and leg on the opposite side flex. With Galant's reflex, the examiner supports the baby in the prone position and then strokes one side of the back about 1 centimeter lateral of midline, causing the baby to curve his or her trunk in the direction of the stimulation. With the parachute maneuver, the examiner holds the baby prone and then lowers the baby's head, prompting the infant to extend his or her arms and legs out like a parachute. This reflex usually does not occur until 8 months or so of age. Other primitive reflexes include rooting, palmar grasp (shown below), plantar grasp (shown below), and stepping (shown on next page). The depiction shows the stepping motion of the child; however, this examination should occur with the leg raising up to a higher surface.

Sciatica

Sciatica typically occurs in the healthy middle-aged adult. Pain is acute in onset and radiates to the leg. Pain starts in the lower back and as it intensifies radiates into the lower extremity. The most common sites of involvement are between L4-L5 and L5-S1 vertebrae (see Figure 8.2). With disc herniation, pain is characterised as being dull, deep and aching. It is usually felt in the upper part rather than the lower part of the leg and spreads from the lumbar spine. If the disc ruptures or herniates under strain then the pain is usually lancinating in quality, shooting down the leg like an electric shock. Valsalva movements, for example coughing, sneezing or straining at stool, often aggravate the pain. Referral is needed for confirmation of the diagnosis. GPs can perform a straight-leg raising test whereby the pain of sciatica can be induced by elevating the leg of the patient when lying down. Prognosis is good, although improvement and recovery is often slower than in low back pain alone.

A 17-year-old boy presents following an injury to his right dominant upper arm while playing football a half hour ago. While attempting to throw a pass, an opposing player grabbed his arm and tackled him. The opposing player landed directly on the thrower's flexed elbow, while driving the thrower's shoulder into the ground.The injured player's mother heard a "cracking sound" in the stands. He had immediate pain and exhibited a deformity in the distal 1/3rd of his right arm. Besides the inspection findings of edema, ecchymosis, and deformity, what specific physical examination technique should be performed?

Since radial nerve injuries can be associated with this fracture, checking the neurological status of the radial nerve by asking the patient to extend his wrist (which should not be painful at the fracture site) and checking the sensory distribution on the dorsal hand is imperative. The axillary nerve is proximal to the fracture and is unlikely to be affected.

A 55-year-old woman presents with a 3-week history of dull pain in the left femur. She does not remember any trauma to her leg. After seeing an abnormality on the X-ray, you obtain a bone biopsy and see poorly-to-well-formed cartilage-like cells with a bizarre appearance. Question What is the most likely diagnosis?

Since there are cartilage-like cells in the biopsy, it is very likely a chondromatous tumor, the widely spread differentiation points to a sarcoma. It predominantly occurs in the pelvis, femur, and shoulder girdle in middle-aged to older adults. It may be primary, arising from cartilage cells, or secondary to a preexisting benign lesion. In osteomalacia (failure to calcify the bone matrix), there would be a decreased amount of well-calcified bone. An old fracture would show unorganized meshwork of woven bone (callus). In Paget's disease, also known as osteitis deformans, there would be dense, well-calcified, erratically shaped bony spicules with abnormal grouped cement lines, which form a mosaic pattern. A bone cyst would show a big, solitary, cystic cavity.

You are evaluating a 32-year-old male who has possibly injured his Achilles tendon while planting his foot into the soft ground while playing touch football. When evaluating a patient with this suspected injury, you have the patient plantar flex his ankle as part of the exam. Based on your knowledge of anatomy, what two plantar flexing muscles attach into the Achilles tendon? A Anterior tibialis and soleus B Flexor hallucis longus and flexor digitorum C Flexor hallucis longus and gastrocnemius D Gastrocnemius and soleus E Peroneus longus and peroneus brevis

The Correct Answer is: D The gastrocnemius and soleus muscles form a common tendon at their distal insertions known as the Achilles tendon. This the strongest and thickest tendon in the body and it attaches at its distal end onto the calcaneus bone of the foot. Contraction of the gastrocnemius and soleus muscles are responsible for plantar flexion of the foot and ankle. The anterior tibialis muscle is one of the important dorsiflexors of the foot and ankle. Flexor hallucis longus and flexor digitorum are both involved in plantar flexion, but are not as strong as the gastrocnemius and soleus muscles and do not attach to the Achilles tendon. The flexor hallucis longus is primarily responsible for plantar flexion of the great toe and flexor digitorum produces plantar flexion of the remaining toes. The peroneus longus and peroneus brevis muscles do contribute to plantar flexion, but are not connected to the Achilles tendon. They are also the primary evertor muscles of the foot and ankle

A 64-year-old woman with a medical history of rheumatoid arthritis and deteriorating vision presents to the clinic with complaints of painful bilateral swelling of her ankles and hands, morning stiffness, loss of appetite, and fatigue. She is currently taking naproxen sodium 500 mg twice per day. Which medication(s) would be most appropriate for this patient? A acetaminophen B azathioprine C cyclosporine D hydroxychloroquine E methotrexate

The Correct Answer is: E Whereas NSAIDs such as naproxen provide some symptomatic relief in rheumatoid arthritis, they do not alter disease progression like DMARDs (disease-modifying antirheumatic drugs). NSAIDs are best used in conjunction with DMARDs. Methotrexate is usually the DMARD of choice because it is well tolerated by the majority of patients and can produce beneficial effects in 2 to 6 weeks. Hydroxychloroquine is another DMARD that can be used for rheumatoid arthritis but can produce ocular toxicity. Cyclosporine and azathioprine are used less frequently today due to toxicity and lack of long-term benefits. Similar to naproxen, acetaminophen would also fail to alter disease progression.

TOC for Ankylosing Spondylitis

The above patient has ankylosing spondylitis and should be treated initially with indomethacin or other nonsteroidal anti-inflammatory drugs (NSAIDs). Ankylosing spondylitis is an autoimmune disorder that leads to progressive stiffening and eventual fusion of the spine Etanercept, a tumor necrosis factor (TNF) inhibitor, may be used in the treatment of ankylosing spondylitis. However, it is considered a second line agent for those whose disease is not responsive to NSAIDs.

You are evaluating a 32-year-old man who presents with an 11-day history of progressive, atraumatic, left hip pain. His PMH is unremarkable with the exception of intravenous drug use. He complains of increasing pain that originally was aggravated with movement, but now he has pain at rest. He is ambulatory with a limp and exam shows no edema, crepitus, rash, or erythema. Pain is reproduced with hip ROM. Distal CMS is fully intact. Vital signs show a BP of 124/93, HR 134, RR 20, Temp 39.4, and O2 sat of 96% RA. Laboratory studies show a leukocytosis and elevated ESR. Question What finding would you expect on his hip X-ray?

The answer is soft tissue swelling with periosteal elevation.1 This patient presents with osteomyelitis of the hip. Osteomyelitis is infection of the bone, with accompanying inflammation and eventual bone destruction. Risk factors for osteomyelitis include IV drug use, implanted devices and hardware, immunocompromised states, open fractures, and chronic joint destruction, such as in patients with rheumatoid arthritis.

A 23-year-old college basketball player twists her ankle while practicing. She explains the injury that is consistent with an inversion mechanism. Based on this history, what ligament would you expect to be the most likely injured in the ankle of this patient?

The anterior talofibular ligament is the first, and often only, ligament damaged in inversion ankle sprains.

A 24-year-old man was playing basketball when he jumped up to shoot a basketball and landed on another player's leg. The patient grabbed his left ankle immediately and screamed in pain. His whole ankle started to swell, and he had difficulty bearing weight. His friends took him to the emergency room. On exam, the patient had tenderness to the tip of the lateral malleolus. Pain is elicited with forced ankle inversion. Talar tilt test and anterior drawer test cannot be assessed due to swelling and pain. X-rays are negative for a fracture. What ligament did the patient most likely sprain?

The anterior talofibular ligament is the only correct choice, as this is the only ligament listed above that is in the lateral aspect of the ankle. The mechanism of injury and physical exam points to the lateral part of the ankle. The other 4 choices compose what is commonly known as the deltoid ligament complex and are all found medially. The patient jumped up to shoot the basketball and most likely had his foot in a plantar flexion; upon landing on another player, the ankle was inverted. The anterior talofibular ligament is the first to be injured and can be the only ligament injured in lateral ankle sprains. However, the stronger force can injure the other 2 ligaments in the lateral aspect of the ankle, the calcaneofibular ligament and posterior talofibular ligament.

Shoulder dislocation commonly involves injury to which nerve?

The axillary nerve is in close proximity to the glenohumeral joint, thus making it vulnerable to injury during a shoulder dislocation.

Brown-Sequard's syndrome.

The characteristic deficits of the Brown-Sequard's syndrome include a motor deficit and deficit of position sense and vibratory sense ipsilateral to the side of the lesion. There is a pain and temperature deficit that is contralateral to the side of the lesion.

Which bone is the most susceptible and most often fractured at birth?

The clavicle is the most common bone broken during childbirth. It often is associated with shoulder dystocia, but clavicular fractures can occur in uncomplicated pregnancies. They are usually of the greenstick variety and heal without complications.

A 42-year-old woman presents with a history of chronic fatigue, pain around her neck, shoulders, and lower back. She is also experiencing chronic headaches and irritable bowel symptoms. Upon physical exam, no abnormal findings were found, with the exception of trigger points that produced pain around areas such as the trapezius, lateral epicondyle of her elbow, and pain around the medial fat pad of her knee. Laboratory findings showed a normal ESR, negative RF factor, and a negative ANA. Question What is the most likely explanation of the findings?

The clinical picture is suggestive of fibromyalgia. This patient presents with symptoms that are found in patients with fibromyalgia, such as common trigger points noted in her physical exam. Additionally, lab findings help rule out other options, as patients with fibromyalgia have normal lab findings. Osteoporosis is not the correct answer, because the patient does not have a T score of less than -2.5. Rheumatoid arthritis is not the correct answer, because the patient has a negative RF factor. Systemic Lupus Erythematous is not the correct answer, because the patient has a negative ANA. Polymyalgia rheumatica is not the correct answer, because the patient has a normal ESR.

A 10-year-old boy presents with a possible arm fracture. While playing baseball at school, he slid into another student and hit his upper arm on the other student's leg. An X-ray of the patient's right arm reveals that he has a fracture at the distal 1/3 of the humerus. You also note that the patient cannot extend his wrist. Question What structure has most likely been injured?

The clinical picture is suggestive of injury to the radial nerve. The characteristic sign of injury to the radial nerve is wrist-drop. Wrist-drop is the inability to extend the wrist and the digits of the metacarpophalangeal joints. Injury to the brachial artery would result in contraction of the digits and sometimes the wrist. Since there is collateral circulation, symptoms of ischemia to the distal nerves and muscles may take up to 6 hours to present. Injury to the musculocutaneous nerve would result in weakening of elbow flexion and supination of the forearm. Injury to the median nerve would result in loss of flexion of the PIP joints in digits 1 to 3. Injury to the ulnar nerve can result in sensory loss in the hand, and patients may have difficulty making a fist because they cannot flex their 4th and 5th digits at the DIP joint.

A 28-year-old woman presents with severe right ankle pain. Further questioning reveals the patient is unable to move her ankle. She states that it is exquisitely tender and the pain worsens with movement or when attempting to bear weight. She states that she cannot walk with the right leg because of how substantially painful it is. During the physical examination, you note that the right ankle is painful to even light palpation. The skin is a healthy color and there are no signs of lacerations or tenting. Pulses are palpable. The entire area of the right ankle is swollen and appears dislocated. The left leg, foot, and ankle are unremarkable. Question Given the history and physical examination, what is this patient most likely suffering from?

The correct answer for this patient scenario is a right ankle fracture. Since there is significant swelling bilaterally, both the tibia and the fibula are very highly likely to be fractured. Key features of an ankle fracture include pain, swelling/signs of bruising after the traumatic event, apparent deformity, and the inability to bear weight; the patient, in this case, fits all of these components.

A 32-year-old golfer stepped out of the golf cart on the eighth hole, twisting his ankle. He has pain in his foot, and radiographs confirm a fracture toward the proximal portion of the fifth metatarsal. Which of the following best describes this fracture? Jones fracture Torus fracture Bennett's fracture Smith's fracture

The correct answer is A. A Jones fracture is a fracture located at the base of the fifth metatarsal at the junction of the diaphysis and metaphysis. A torus fracture is a compression fracture of the cortex occurring 2 to 3 cm proximal to the physis and is most common in the distal radius of a child. A Bennett's fracture is an oblique intra-articular fracture occurring at the base of the first metacarpal. A Smith's fracture is a displaced volarly angulated (apex dorsally) extra-articular fracture of the distal radius.

26. A 62-year-old white male, whose father is from Norway, presents to your office with a chief complaint of hand pain and some functional deficit (difficulty extending his finger). Exam reveals a palpable thickening which resembles a callus in a cordlike presentation. A picture of his problem is shown below. Which of the following best describes what this patient is developing? Save Image Enlarge Image Dupuytren's contracture Carpal tunnel syndrome Trigger finger De Quervain's tenosynovitis

The correct answer is A. Dupuytren's disease is a nodular thickening and contraction of the palmer fascia. It causes hard nodules near the distal palmar crease, which eventually result in contracture of the finger at the metacarpal phalangeal joint. It most commonly affects men older than age 50 and has a dominant genetic component involving people of Northern European descent, which is why it is sometimes referred to as "Viking disease." Carpal tunnel syndrome is median nerve compression (entrapment) at the wrist; it is most common among middle aged and pregnant women. Trigger finger results when the first annular pulley in the volar palm of the hand becomes thickened and stenotic from chronic inflammation and irritation. Motion of the tendon is limited, and the finger may snap or lock during flexion or may be unable to extend actively. De Quervain's tenosynovitis is swelling or stenosis of the sheath that surrounds the abductor pollicis longus and the extensor pollicis brevis tendons on the thumb side of the wrist. The inflammation thickens the tendon sheath and constricts the tendon as it glides in the sheath. This can cause pain. Often precipitated by repetitive use of the thumb, it is more common in middle-aged women.

A 21-year-old volleyball player comes to the emergency department after twisting her ankle while playing volleyball. She complains of pain over the outside of her ankle and swelling. She can walk, but only with pain. On physical exam, she has pain on inversion of the ankle and palpable tenderness just inferior to the lateral malleolus. On ligament testing with inversion, she has slight joint laxity with pain. Which of the following is the most likely diagnosis? Moderate or grade II lateral ankle sprain Deltoid ligament tear Distal tibial fracture Retrocalcaneal bursitis

The correct answer is A. Lateral ankle sprains are the most common. A sprain of the deltoid ligament is a sprain of the medial ankle ligaments. If it were a tibial fracture, the patient would be unable to walk.

Hurts to cross arm over body

The correct answer is A. Pain with adduction of the arm constitutes a positive "crossover test" and indicates inflammation or degenerative changes in the acromioclavicular joint.

A 61-year-old female with a long-standing history of cervical neck arthritis presents to your office with complaints of paresthesias in the second and third fingers of her left hand. She admits this has been present for 6 to 8 months. Which test would clinically confirm a diagnosis of carpal tunnel syndrome versus a cervical radiculopathy? Phalen's test Finkelstein's test Froment's test Grip strength

The correct answer is A. Phalen's test evaluates for carpal tunnel syndrome. The patient holds both forearms perpendicular to the ground, flexes both wrists, and then holds them against each other for 30 seconds. Patients who experience numbness or tingling in the distribution of the median nerve have a positive result. Finkelstein's test evaluates for De Quervain's tenosynovitis of the wrist. The performance of this test entails flexing and ulnarly deviating the wrist while the thumb is grasped into flexion. Pain at the dorsoradial aspect of the wrist indicates tenosynovitis of the first dorsal compartment (abductor pollicis longus and extensor brevis) tendons. Froment's test evaluates for weakness or paralysis of the flexor pollicis brevis and impairment of the conduction of the median nerve supplying muscles to the thenar eminence. In attempting to grasp a playing card between the thumb and base of the index finger, the flexor pollicis longus takes over from the weakened pollicis brevis so that the thumb collapses into flexion at the interphalangeal joint. Grip strength can be diminished in both cervical radiculopathy or carpal tunnel.

A 45-year-old male presents limping into your office. He tells you that he was playing basketball with his kids yesterday and felt a "pop" near his knee when he jumped up to get a rebound. He immediately felt pain and had a hard time with ambulation afterward. On physical exam, he is unable to extend the knee fully, has swelling at his proximal knee, and has a palpable defect at his suprapatellar region. Which of the following is the most likely diagnosis? Quadriceps tendon rupture Knee dislocation Achilles tendon rupture Chondromalacia patella

The correct answer is A. Quadriceps tendon ruptures most often occur in patients older than 40 years of age. They are common with jumping activities. Knee dislocations are uncommon. Achilles tendon ruptures occur more distally. Chondromalacia patella is a more chronic problem.

Which of the following is a vitamin D deficiency resulting in abnormalities in the shape and structure of bones? Rickets Scurvy Bulimia Beriberi

The correct answer is A. Rickets is a vitamin D deficiency in children that results in inadequate deposition of lime salts in developing cartilage and newly formed bone, causing abnormalities in the shape and structure of bones. Scurvy results from a vitamin C deficiency, usually due to a dietary lack of fruits and vegetables. It presents with hemorrhagic manifestations and the abnormal formation of bones and teeth. Bulimia nervosa is a disorder marked by episodes of binge eating followed by self-induced vomiting and diarrhea, excessive exercise, strict dieting or fasting, and an exaggerated concern about body shape and weight. Beriberi results from a thiamine deficiency. This disease presents with peripheral neurologic, cerebral, and cardiovascular abnormalities. Early deficiency produces fatigue, irritation, poor memory, and sleep disturbances. It is endemic in Asia and the Philippines due to a diet high in polished rice that has lost all thiamine content through the milling process.

An 8-year-old boy who plays multiple sports complains of posterior heel pain in his right foot. Which of the following is the most likely diagnosis? Sever's disease Kohler's disease Freiberg's infraction Charcot-Marie-Tooth disease

The correct answer is A. Sever's disease, also known as calcaneal apophysitis, most commonly affects active prepubertal children and presents with pain in the posterior aspect of the heel following play and sports activities. Kienbock's disease is osteochondrosis of the lunate bone of the wrist. Freiberg's infraction is osteochondritis of the head of the second metatarsal bone of the foot. Charcot-Marie-Tooth is a form of progressive neural atrophy of muscles supplied by the peroneal nerves. It presents as progressive weakness of the muscles of the feet. Muscles atrophy, reflexes are lost, foot drop develops, and cutaneous sensations are lost. The disease usually develops in childhood and is more common in males.

34. Which of the following statements is false regarding Lyme disease? Cranial nerve palsy can present. Common rash is erythema multiforme. The knee is the most commonly affected joint should arthritis develop in an untreated patient. The disease is named after a town in Connecticut.

The correct answer is B. Erythema migrans is the rash related to Lyme disease, which usually presents followi ng a bite from an infected tick. The rash has an inflamed annular border and moves away from the site. It eventually becomes a large, bluish-red area with a firm border. Neurologic symptoms of Bell's palsy (unilateral facial paralysis affecting the seventh cranial nerve) can develop in a patient with Lyme disease. Arthralgia and arthritis occur in up to 80% of untreated patients with Lyme disease. The knee is the most commonly affected joint. Lyme disease received its name from the town of Lyme, Connecticut, in 1975 after several children developed a mysterious arthritis of unknown cause that was subsequently found to be caused by the spirochete Borrelia burgdorferi. Erythema multiforme is a macular eruption with dark red papules, wheals, vesicles, and bullae. It typically occurs on the extremities, including the palms and the soles. It appears in successive eruptions for short durations. The eruption may appear in separate concentric rings called target lesions. The cause is presumed to be an immune reaction to antigens, such as viruses or mycobacteria, or as a side effect to certain drugs. Stevens-Johnson syndrome is a severe form of erythema multiforme in which there are bullae on the oral mucosa, pharynx, conjunctiva, and anal area. The rash does not relate to Lyme disease.

An 18-year-old field hockey player stopped to reverse her direction as the play transitioned from offense to defense. She felt a "pop" as her knee buckled and developed swelling. Aspiration of her knee yielded a hemarthrosis. An anterior cruciate ligament (ACL) tear is suspected. Which of the following exam maneuvers would be positive for an ACL tear? Straight-leg raise Lachman's test McMurray's test Posterior drawer test

The correct answer is B. Lachman's test evaluates the anterior cruciate ligament (ACL) for deficiency. With the thigh supported and thigh muscle relaxed, flex the knee to 15°. Grasp the distal femur from the lateral side with one hand and the proximal tibia from the medial side with the other hand. Initiate a shucking motion by pulling anteriorly on the tibia while pushing posteriorly on the femur. Focus on the amount of bony translation of the tibia relative to the femur. Increased anterior translation indicates a tear of the ACL. The straight-leg raise tests for competency of the quadriceps and patella tendon. If a patient is unable to perform a straight-leg elevation while lying in the supine position, the integrity of either extensor mechanism is in question. McMurray's test evaluates for meniscal tears. For a questionable medial meniscus tear, flex the knee of a patient who is lying in the supine position. With the knee flexed, externally rotate the tibia on the femur. Gradually extend the knee. If this maneuver elicits a click or pain, the clinician should be suspicious of a posterior horn medial meniscus tear. The posterior drawer test helps to elicit posterior cruciate ligament (PCL) deficiencies. With the patient supine, flex the knee to 90°. With a PCL injury, the tibia is in a posterior position relative to the femur.

A 31-year-old male is playing volleyball. He strikes the tip of his second finger of his right hand on the ball when trying to block an opponent's shot. He has pain at the distal interphalangeal joint and decreased motion. Radiographs show a small avulsion fracture of the proximal aspect of the distal phalanx. This is also associated with the patient being unable to extend his finger. The radiograph is shown below. Which of the following best describes this patient's injury? Save Image Enlarge Image Boutonnière deformity Mallet finger Dupuytren's contracture Trigger finger

The correct answer is B. Mallet finger deformities occur due to disruption of the terminal extensor tendon at the distal interphalangeal joint, causing an active extension deficit. Boutonnière deformity can occur as a result of progressively worsening arthritis or secondary to an acute trauma. The cause is due to damage of the central slip, which is vital to the extensor mechanism of the proximal interphalangeal (PIP) joint. It results in a fixed flexed PIP joint and a hyperextended distal interphalangeal joint (DIP) joint. Dupuytren's contracture results from a nodular thickening and contraction of the palmar fascia, causing an inability to extend the finger. Trigger finger refers to the first annular pulley in the volar palm of the hand, which may become thickened and stenotic from chronic inflammation and irritation. Motion of the tendon is limited, and the finger may snap or lock during flexion or may be unable to actively extend the finger.

42. A 13-year-old boy just finished his soccer season and is now playing basketball in school. He has pain in the front of his knees (right greater than left) that worsens with activities and improves with rest and ice. Which of the following is the most likely diagnosis? Gaucher disease Osgood-Schlatter disease Legg-Calvé-Perthes disease Reiter syndrome

The correct answer is B. Osgood-Schlatter disease (OSD) results from repetitive injury and small avulsion injuries at the patellar tendon insertion into the secondary ossification center of the tibial tuberosity. Gaucher disease is a chronic disorder of lipid metabolism caused by a deficiency of the enzyme beta-glucocerebrosidase. Legg-Calve-Perthe's disease is an idiopathic avascular necrosis of the hip epiphysis of the femoral head and its associated complications in a growing child. Reiter syndrome is a syndrome consisting of urethritis, arthritis, and conjunctivitis. It occurs mainly in young men. Chlamydia trachomatis is the organism most frequently isolated.

A 73-year-old woman was watering her lawn when she tripped over the hose, landing on the ground with most of the impact on her right hyperflexed hand. Her radiograph is shown below. Which of the following statements most appropriately applies to this fracture? If the fracture is stable, it can be casted without trying to reduce the displaced fracture. The complications of such a fracture will be associated with median nerve or radial artery damage. The common name for this fracture is Colles' fracture. The most appropriate treatment for this fracture is to wrap a bulky dressing around the injured area until the swelling subsides and then casting the patient in a long arm cast for 6 weeks.

The correct answer is B. Smith's fracture is a displaced volarly angulated (apex dorsally) extra-articular fracture of the distal radius (wrist). Potential complications with this fracture presentation include median nerve damage and radial artery damage. A displaced fracture needs anatomical realignment. It should not be casted as is. Colles' fracture is a displaced dorsally angulated (apex volar) extra-articular fracture of the distal radius (wrist). Again, a displaced fracture needs anatomical realignment. The treatment plan should not solely be a bulky dressing until the swelling subsides and then a long arm cast; the fact that the fracture is displaced and needs to be reduced must be addressed.

25-year-old man presents with back pain and stiffness. He states he has had longstanding issues with back pain, although he denies any trauma to his back. He has noticed along with the pain an increasing presence of stiffness and general fatigue. He feels that these issues have gradually worsened over the last several months; they are more persistent recently. He notes that the pain is much worse first thing in the morning, rating it a 6 - 7/10 on a pain scale. Radiation occasionally occurs into the buttock areas and the patient feels the symptoms actually lessen with activity. Physical examination shows marked forward stooping of the thoracic and cervical spine with the lower spine showing the presence of a substantial reduction in lateral flexion. Question What would be the first-line pharmaceutical treatment to consider in this patient?

The correct response is NSAIDs. The patient history and physical examination described above is most likely a case of ankylosing spondylitis (AS). This is a chronic inflammatory disease that consists of many signs of symptoms, specifically significant back pain and progressive spinal stiffness. Many times, patients with AS will also have transient or persistent peripheral arthritis as well as other manifestations such as anterior uveitis. Typically this is seen in male patients who are in the age range of 20 - 30 years old, although some may start having issues as early as their late teens. TNF inhibitors and etanercept are both viable secondary options for patients who have failed relief from NSAID trials, but they are not considered first-line treatment. Sulfasalazine is sometimes used to help relieve symptoms in peripheral arthritic manifestations, but it is not useful for spinal or sacroiliac disease in AS. Corticosteroids have a very minimal impact on treating AS, and they could even worsen the osteopenia that is sometimes seen in this pathology.

A 65-year-old woman presents with morning stiffness and pain in her shoulders. She reports that she has lost a few pounds over the past month and has had intermittent fevers. The physical examination reveals tenderness over her deltoids with reduced shoulder abduction.

The correct response is an elevated C-reactive protein. The clinical presentation is suggestive of polymyalgia rheumatica; patients usually present with pain and stiffness of the pectoral and pelvic girdles. They may also have constitutional symptoms, such as weight loss and fever. On examination, the affected shoulder or thigh muscles are tender, and there is a painful restriction of hip and shoulder movements.

A 64-year-old woman presents with malaise and severe unilateral headache, as well as pain and stiffness in her neck, shoulders, and back. Her appetite is poor, and she has recently lost weight. She has an oral temperature of 100.5° F, her hemoglobin is 11.8 g/dL, and the sedimentation rate is 104 mm/hr. Question If temporal arteritis were to be demonstrated in this patient, of what would therapy consist?

The correct response is corticosteroids. Giant cell (cranial/temporal) arteritis (GCA) is associated with polymyalgia in 25% of people. Common in the elderly, it is rare under 55 years. The symptoms include headache, scalp, temporal artery tenderness (e.g., when combing hair), jaw claudication, amaurosis, and fugax. A decrease or loss of temporal artery pulsations may occur in patients with cranial or temporal arteritis.

A 22-year-old Asian man with no significant past medical history presents with a dull pain. It is insidious in onset and has been felt deep in the lower lumbar gluteal region for the past 5 months. It is accompanied by low-back morning stiffness of up to a few hours' duration. The stiffness improves with activity and returns following inactivity. For the past 2 months, the pain has ascended; it has become more persistent and bilateral. He notes that nocturnal exacerbation of pain forces him to rise and move around. He also admits to bilateral hip and ankle pain, anorexia, malaise, and an intermittent low-grade fever. His physical exam reveals a stooped forward-flexed position, limitation of anterior and lateral flexion and extension of the lumbar spine, a positive Schober test, restricted ranges of motion of his hips, and reduced chest expansion. What additional clinical manifestation would be most likely in this patient?

The correct response is eye pain and conjunctival injection. This patient's most likely diagnosis is ankylosing spondylitis (AS). The most common extra-articular manifestation is acute anterior uveitis, which occurs in 40% of patients and can antedate the spondylitis. Attacks are typically unilateral, causing pain, photophobia, and increased lacrimation. These tend to recur, often in the opposite eye. Cataracts and secondary glaucoma are not uncommon sequelae. Up to 60% of patients have inflammation in the colon or ileum. This is usually asymptomatic, but frank IBD occurs in 5 - 10% of patients with AS. A positive straight leg raise test indicates a likelihood of her herniated lumbar disc.

A 65-year-old woman presents after tripping on a rug in her home and falling on her outstretched arm. She states that her "hand bent backwards" and she heard a "snap". Physical exam shows an obvious deformity over the distal radius, with radial shortening. What anatomical structure may also be injured by this classic Colles' fracture?

The correct response is median nerve. Due to the mechanism of the injury (i.e., forced wrist dorsiflexion), the distal fragment is angulated dorsally, and the proximal fragment is volarly displaced. Bony fragments or compression within the carpal tunnel may injure the median nerve.

A 4-year-old girl presents 30 minutes after falling from a slide (about 4 feet fall) on her outstretched right dominant arm with her elbow fully extended. There was no loss of consciousness, but there was immediate extreme pain; there is an obvious deformity at the elbow. On presentation, the girl is holding her right elbow, and she will not allow passive movement. Question What finding would be expected on exam?

The correct response is severe pain with flexion of the elbow. This is the classic age and mechanism of injury for a supracondylar humerus fracture. Pain is worse with flexion or extension, especially in displaced fractures (as indicated by the deformity in this child's case). The axillary nerve would not be injured in this case; it is proximal to the elbow and rarely involved.

A 25-year-old woman presents with pain and tingling sensation in her right hand. Her symptoms are especially intense at night. By holding her hand flexed for about a minute, you can provoke the symptoms. The paresthesia is extended to the palmar area of the thumb, index and middle finger, as well as half of the ring finger; thumb adduction and apposition are weakened on the right. What nerve is affected?

The described symptoms are typical for carpal tunnel syndrome, a compression of the median nerve in the volar aspect of the wrist between the flexor tendons and the superficial transverse ligament. The syndrome is relatively common, affects more women than men, can be uni- or bilateral, and is frequently associated with occupations that require a lot of wrist flexion. Compression of the ulnar nerve is often caused by trauma. Repeatedly leaning on the elbow is an example of trauma that could cause it. The paresthesia affects the palmar area of the 5th and half of the 4th finger as well as the dorsal area of the 5th, 4th, and half of the 3rd finger. There can also be weakness of the thumb adductor, 5th finger abductor, and interossei muscles. Compression of the radial nerve (having an arm hang over the back of a chair for a long period of time is an example of something that could cause it) leads to paresthesia over the area of the first interosseus muscle and weakness of wrist and finger extensors. The musculocutaneous nerve supplies the upper arm flexors and the skin of the radial side of the forearm. Compression of the peroneal nerve in the area of the fibular neck is common in bedridden patients or those with badly administered leg casts. Dorsiflexion and eversion of the foot are weakened, and a sensory deficit can be found over the anterolateral area of the lower leg, the dorsum of the foot, or between the 1st and 2nd metatarsal.

A 15-year-old boy presents for a routine physical. He reveals a 1-month history of mildly painful swelling of the anterior superior left shin. It has been unaccompanied by fever, erythema, or joint complaints. He is his high school football team's tailback. He has been assisting his father in caulking the hull of their new boat, which entailed considerable kneeling that worsened the pain in the affected area. Examination is unremarkable except for mild, slightly tender swelling of the left anterior shin approximately 5 cm below the knee. Radiographs of the left knee showed mild irregularity of the tibial tubercle. Question What treatment recommendation is appropriate in this case?

The diagnosis is Osgood-Schlatter disease (condition), which is an inflammation of the tibial tuberosity. Once the significant conditions are eliminated, treatment is supportive and symptomatic.

What is the first type of motion that is lost as a person develops progressively worsening osteoarthritis of the hip joint?

The earliest sign of the development of osteoarthritis of the hip is often the loss of internal rotation. Also, the gluteus medius (which is a hip abductor and helps stabilize the pelvis) may become weakened as the condition worsens, resulting in an abductor lurch as the trunk of the body sways out over the affected limb when attempting to walk

A disk herniation that is putting pressure on the L5 nerve root may present with weakness of what muscle(s)?

The extensor hallucis longus muscle's motor function is associated the L5 motor neuron, which also supplies the gluteus medius and extensor digitorum longus and brevis muscles. The anterior tibialis muscle is supplied by the L4 motor neuron. Nerves emanating from T12, L1, L2 and L3 supply the iliopsoas. Gastrocnemius, soleus and peroneus longus and brevis are all supplied by nerves coming from the S1 area. The plantar flexing gastrocnemius and soleus muscles also are supplied by S2.

A newborn boy was observed to have a small cyst located midline in the lumbar region of his back. No neurological symptoms were observed, and the sac contained only meninges and cerebral spinal fluid. X-ray revealed the absence of a vertebral arch at L5. What is this type of anomaly?

The failure of the 2 halves of the vertebral arches to fuse during development results in an anomaly referred to as spina bifida. The most minor, clinically insignificant type (spina bifida occulta) is asymptomatic and may be revealed only with imaging of the vertebra involved. Sometimes a small dimple or tuft of hair overlying the defect may be present at the level of the malformation, usually in the lumbar or sacral regions. More serious anomalies, categorized as spina bifida cystica, show a cyst-like protrusion at the level of the defect. When the cyst or sac contains meninges and cerebrospinal fluid, it is referred to as spina bifida with meningocele. If the spinal cord and/or nerve roots are included with the meninges and CSF in the sac, it is referred to as spina bifida with meningomyelocele.

A 34-year-old man presents with a 10-year history of progressive weakness in his arms and legs. His problems include difficulty brushing his hair and raising his arms about his head. He also is unable to rise from a chair without using his arms to push himself up from a seated position. Your exam notes decreased muscle bulk and objective weakness of the deltoid, biceps, triceps, hip flexors, and hamstrings. Hand strength and handgrip release is normal. He reports that his father developed similar symptoms as a young man, and the father now requires a wheelchair. Question What diagnosis best explains the patient's symptoms?

The history and physical exam are most compatible with a limb-girdle muscular dystrophy that affects the proximal muscle of the shoulder and pelvic girdles. The condition is not usually present at birth, but develops in young adults. The symptoms are gradual and progress over many years, and some adults will end up requiring assistive devices. Multiple-inheritance patterns have been described, including the autosomal dominant inheritance that best fits this case. Duchenne muscular dystrophy has an onset in young boys, progresses to death (usually in the late teenage years), and has an X-linked inheritance pattern where male-to-male transmission is not possible. Hemochromatosis is a disorder of iron overload that does not typically cause overt or prominent muscular symptoms. Instead, complains of arthritis and fatigue are common. Hepatic dysfunction is common in latter stages of the disease.

A 48-year-old woman presents with a chief complaint of gradually progressing difficulty in climbing stairs over the past 3 months. The physical examination shows there is notable proximal muscle weakness of the upper and lower extremities. The remainder of the examination is unremarkable. The laboratory evaluation shows an elevated serum creatinine phosphokinase level, and a muscle biopsy reveals lymphoid inflammatory infiltrates. Which of the following is the appropriate initial treatment of choice in this patient?

The most likely diagnosis in this patient is polymyositis. This is supported by the finding of a gradual progressive proximal muscle weakness and elevation of creatinine phosphokinase level. The finding of lymphoid inflammatory infiltrates on muscle biopsy confirms the diagnosis. Initial treatment of choice in this condition is the use of a corticosteroid (prednisone).

A 48-year-old woman presents with a chief complaint of gradually progressing difficulty in climbing stairs over the past 3 months. The physical examination shows there is notable proximal muscle weakness of the upper and lower extremities. The remainder of the examination is unremarkable. The laboratory evaluation shows an elevated serum creatinine phosphokinase level, and a muscle biopsy reveals lymphoid inflammatory infiltrates. Which of the following is the appropriate initial treatment of choice in this patient?

The most likely diagnosis in this patient is polymyositis. This is supported by the finding of a gradual progressive proximal muscle weakness and elevation of creatinine phosphokinase level. The finding of lymphoid inflammatory infiltrates on muscle biopsy confirms the diagnosis. Initial treatment of choice in this condition is the use of a corticosteroid (prednisone). Patients who do not respond to prednisone may then benefit from the use of methotrexate or azathioprine. Both intravenous immune globulin and hydroxychloroquine are effective for the treatment of patients with dermatomyositis that is resistant to prednisone therapy

A 67-year-old man presents with pain and stiffness in his shoulders and hips lasting for several weeks with no history of trauma. He also has complaints of headache, throat pain, and jaw claudication. It is imperative to diagnose this patient promptly in order to prevent which of the following complications? A anemia B cerebral aneurysms C mononeuritis multiplex D ischemic optic neuropathy E respiratory tract complications

The most urgent need for diagnosis of a patient with symptoms of polymyalgia rheumatica (PMR) and giant cell arteritis is to prevent blindness caused by ischemic optic neuropathy as a result of occlusive arteritis of the ophthalmic artery. Early diagnosis is imperative as the neurological damage to the optic nerve is not reversible. Most patients with this diagnosis will have a normochromic-normocytic anemia, but this does not create urgency in treatment. Cerebral aneurysms are not common findings with PMR; large vessels such as the subclavian and aorta may be involved in giant cell arthritis in 15% of patients. Mononeuritis multiplex commonly presents with painful paralysis of a shoulder, and respiratory tract complications are more nonclassic findings with the presentation of PMR

pain. She states that she was playing with her 2 children in their backyard over the weekend and fell onto solid ground, landing directly on her knees. She noted immediate and significant right knee pain. She notes significant swelling; it accompanies the knee pain which she rates as an 8/10 on a numerical pain scale. Physical examination reveals significant obvious joint effusion and exquisite focal tenderness to palpation over the patellar area of the right knee; the left knee has no obvious abnormalities. She is able to appropriately extend both legs against gravity. Results of the anterior drawer, McMurray, and varus/valgus stress testing are within normal limits. Question Given these history and physical examination findings, what is the most likely diagnosis?

The patient above has most likely experienced a right patellar fracture. Many fractures of the patella are the result of direct forces, such as striking the dashboard in a motor vehicle accident, or in our patient's scenario, falling onto a flexed knee. Patellar fractures should be high on one's differential diagnosis list if a patient presents with acutely swollen knee and patella pain following trauma to that area. There will be presence of jointeffusion and focal tenderness to palpation of the patella. Another feature is that the patient will be unable to extend the knee against gravity, which requires surgical repair if present. Our patient had all these signs and symptoms, minus the inability to extend the knee.

A 39-year-old male patient presents with low back pain with radiation to the right leg. On examination you place the right hip in a flexed position, and as you palpate between the iscial tuberosity and the greater tronchanter of the femur the patient complains of radiation of pain down his right leg. Based on this history and exam, which nerve was affected by this part of the examination maneuver?

The sciatic nerve does lie midway between the ischial tuberosity and greater trochanter and it can be palpated when the patient is in a hip flexed position. The gluteus maximus obscures the nerve from being effectively palpated when the leg is in an extended position. Tenderness of the sciatic nerve can be caused by a lumbar disk herniation, direct trauma, or spasm of the nearby pyriformis muscle. The femoral nerve is a deep structure that lies lateral to the femoral artery and is not considered to be palpable. The femoral nerve is responsible for the L1-3 dermatomes and for supplying motor function to the iliopsoas muscle. The peroneal nerve originates from the sciatic nerve and splits into the superficial and deep peroneal nerves, which are responsible for much of the sensory and motor nerve function in the lower leg. The saphenous nerve originates from the femoral nerve in the femoral triangle and runs down the medial aspect of the leg. The sural nerve has medial and lateral components that are found in the lower leg. The medial cutaneous sural nerve arises from the tibial nerve just below the knee and eventually connects with peroneal nerve to form the sural nerve. On the lateral side of the lower leg, the sural nerve arises from the common peroneal nerve just above the knee and eventual connects with the previously discussed medial branch to form the sural nerve.

As you perform the physical examination, what part of the body would exhibit sensation for the C7 component of the brachial plexus?

The sensory nerve emanating from the C7 level is responsible for sensation of the third finger. The lateral forearm is supplied by C6. The lateral upper arm is supplied by C5. The medial upper arm is supplied T1 and the medial forearm is supplied by C8.

straight-leg elevation of quads

The straight-leg raise evaluates the competency of the quadriceps and patella tendon. If a patient is unable to perform a straight-leg elevation while lying in the supine position, the integrity of either extensor mechanism is in question

Duchenne muscular dystrophy

This X-linked condition is maternally transmitted to male children only. It usually manifests in the first decade of life as difficulty in standing and walking. Muscle weakness initially appears in the muscles of the hip girdle and upper legs. Later, the condition involves muscles of the arms and respiratory system, with death usually occurring before age 20. Patients fail to produce the protein dystrophin. This is a member of the spectrin superfamily, with a MW = 427,000. Dystrophin is found underneath the sarcolemma in normal muscle, especially where membrane folds are formed, e.g., at myotendinous and neuromuscular junctions. It also interacts with actin filaments, stabilizing their side-side association as well as binding to the amino-terminus. Muscle from DMD patients shows myocytes of variable diameter (compared to the diameters of b and c) and unusual infiltration of leukocytes (at a). Physical symptoms of DMS occur most often before age 6 and include fatigue, learning difficulties and muscle weakness. It begins most commonly in the legs and pelvis area with progressive difficulty walking. Muscles of DMD patients are more fragile and more easily ruptured. In addition, DMD patients show reduced levels of enzymes normally found in the sarcoplasm with concomitant increases in serum levels of creatine phosphokinase (CPK), lactate dehydrogenase, and glucose phosphate isomerase. Urinary levels of creatine are usually elevated. One of the normal functions of dystrophin is stabilization of the sarcolemma against the physical stresses caused by frequent folding during cycles of contraction and relaxation.Myosin and myoglobin levels would be reduced due to loss on sarcomeres. Lysosomal hydrolases would be elevated because of increased activity from phagocytic cells.

A 19-year-old boy presents with pain and deformity of his right dominant shoulder after a sudden jerking movement to the same from a wrestling competitor approximately 1 hour ago. He was unable to continue wrestling and has pain with any movement of the right shoulder. On exam you see a loss of normal shoulder contour anteriorly. There is no Acromioclavicular joint tenderness. What is the most likely diagnosis?

This is a glenohumeral dislocation (shoulder dislocation). With an acute dislocation there is considerable pain with any movement. There is loss of normal shoulder contour (more so with anterior dislocation rather than posterior dislocation). With a cuff tear there is gross weakness to resistance and gravity. Shoulder (AC) separation would have tenderness over the injury site as would a clavicle fracture or sternoclavicular subluxation.

A 47-year-old grossly obese woman presents with left non-dominant shoulder pain and limited motion. The pain began about a month ago, and her shoulder has progressively lost motion during that time until now she cannot reach overhead with that arm. There is no history of trauma; it does not wake her at night; and she can sleep on the left side. Her past medical history is significant for Diabetes Type I and hypothyroidism. She is on regular insulin and NPH insulin, as well as levothyroxine (Synthroid). What would you expect to find on exam?

This is adhesive capsulitis (frozen shoulder). Diabetes and hypothyroidism are common risk factors.

A 54-year-old man presents with a 2-month history of left non-dominant shoulder pain. There is no history of trauma, but the pain began about a week after shoveling wet heavy snow from his 100-foot driveway. At first the pain seemed to come and go from day to day, but it has gotten progressively worse and more constant. Pain is worse with overhead use and he can't sleep on his left side and will wake up if he rolls over onto his left shoulder. What diagnostic tool can be utilized to help with the diagnosis confirmation?

This is rotator cuff tendonitis/subacromial bursitis/impingement syndrome. A simple subacromial injection of 10 ml of 1% Xylocaine will numb the inflamed tendon/bursa and strength will be normal again as the injected anesthesia takes effect and decreases the associated pain of inflammation (if no cuff tear is associated with it). Plain radiographs are usually normal in any of these disorders. MRI may help rule out a cuff tear but is less sensitive for tendonitis. CT is good for bony pathology mostly and less with soft-tissues. EMG is used for nerve study.

A 32-year-old woman with no significant past medical history presents with a 3-month history of right anterior knee pain which is described as a dull and aching pain that is 'right under the kneecap'. Provocative activities include bending movements, descending stairs, and performing squatting maneuvers. Pain is relieved during rest. She notes that she is an avid outdoorswoman whose hobbies include running and hiking; her symptoms began following a running session. She denies a history of falls, prior surgeries or instrumentation, fever, chills, malaise, myalgias, changes in weight, joint swelling, skin changes or rashes, or other joint pains. Her physical exam is normal with the exception of a tender undersurface of the patella, with crepitus upon passive range of motion of the right knee. There is abnormal patellar tracking upon right knee flexion and apprehension of the patient upon passive manipulation of the patella. Additionally, there is a positive patellar grind test. McMurrary's, Lachman, the anterior and posterior drawer, Apley's compression and distraction tests, and varus/valgus tests are all negative. There is no joint line tenderness, effusion, or restriction of range of motion of the right knee. Question What is the most likely diagnosis?

This patient exhibits signs and symptoms of patellofemoral pain syndrome, also known as chondromalacia patellae, miserable malalignment syndrome, and runner's knee. This syndrome describes any pain involving the patellofemoral joint. The pain affects any or all of the anterior knee structures, including the medial and lateral aspects of the patella, as well as the quadriceps and patellar tendon insertions. The patella engages the femoral trochlear groove with approximately 30 degrees of knee flexion. Forces on the patellofemoral joint increase up to 3 times body weight as the knee flexes to 90 degrees (e.g., climbing stairs), and 5 times body weight when going into full knee flexion (e.g., squatting). Abnormal patellar tracking during flexion can lead to abnormal articular cartilage wear and pain. When the patient has ligamentous hyperlaxity, the patella can sublux out of the groove, usually laterally. Patellofemoral pain is also associated with muscle strength and flexibility imbalances, as well as altered hip and ankle biomechanics. Patients usually complain of pain in the anterior knee with bending movements and less commonly in full extension. Pain from this condition is localized under the kneecap but can sometimes refer to the posterior knee or over the medial or lateral inferior patella. Symptoms may begin after a trauma or after repetitive physical activity, such as running and jumping. When maltracking, palpable and sometimes audible crepitus can occur. Intra-articular swelling usually does not occur unless there are articular cartilage defects or if osteoarthritis changes develop. Patellar mobility can be assessed by medially and laterally deviating the patella (deviation by one-quarter of the diameter of the kneecap is consider normal; greater than one-half the diameter suggests excessive mobility). The apprehension sign suggests instability of the patellofemoral joint and is positive when the patient becomes apprehensive when the patella is deviated laterally. The patellar grind test is performed by grasping the knee superior to the patella and pushing it downward with the patient supine and the knee extended, thus pushing the patella inferiorly. The patient is asked to contract the quadriceps muscle to oppose this downward translation, with reproduction of pain or grinding being the positive sign for chondromalacia of the patella. The Iliotibial band syndrome typically presents with a lateral "snapping" with flexion and extension of the knee. Patients with ACL tears hear an audible "pop" and usually fall down following the injury. They also have acute swelling, difficulty with weight-bearing, and complain of instability. Positive Lachman and Anterior Draw tests are observed. Medial collateral ligamentous injuries usually occur as a result of a valgus stress to the partially flexed knee. It can also occur with a blow to the lateral leg. The MCL is commonly injured with acute ACL injuries. The patient may have a limited range of motion due to pain, especially during the first 2 weeks following the injury. A positive valgus stress test is expected. Injuries to a meniscus can lead to pain, clicking, and locking sensation. Most meniscus injuries occur with acute injuries (usually in younger patients) or repeated microtrauma, such as squatting or twisting in older patients. Clinical findings include an antalgic gait, difficulty with squatting, catching, or locking of the meniscal fragment, effusion, and joint line tenderness. Patients can usually point out the area of maximal tenderness along the joint line. The McMurray test is usually present.

A 70-year-old man with a history of type II diabetes presents with a 2-day history of a red, hot, painful, and swollen left toe. He cannot recall any recent injury or illness, but states that he has been eating a lot of protein-rich foods and consuming alcohol against medical advice. His only complaint is the painful foot. He denies fever, chills, headache, or problems with his right foot. What is the most likely initial treatment for the patient?

Tophi, which are collections of uric acid crystals in the soft tissues, occur frequently in untreated patients. They can be found in multiple locations, including the fingers, toes, and olecranon bursae; they can also be found along the olecranon, where they may appear to be rheumatoid nodules. NSAIDS are the treatment of choice in most patients without underlying health problems. They have an anti-inflammatory effect that works by inhibiting cyclo-oxygenase, which acts to produce leukotrienes from arachidonic acid. Indomethacin is the drug of choice and is usually given at a dose of 25 - 50 mg, 4 times a day, unless the patient is elderly. However, other NSAIDs, such as ibuprofen, naproxen, sulindac, and ketoprofen, may be used. Aspirin is not recommended because it can elevate uric acid levels. Some of the side effects of NSAID therapy include gastropathy, nephropathy, and liver dysfunction. It can also cause fluid overload in patients with congestive heart failure. Colchicine is an antimitotic drug that is often used in patients who cannot take NSAIDs or corticosteroids. Allopurinol is used in the prevention of future attacks. It is not used to treat acute flare-ups. Patients with frequent gout attacks may begin prophylactic treatment to prevent further episodes. Allopurinol is used in such cases. Allopurinol blocks xanthine oxidase, thereby reducing the production of uric acid. It should be used in patients who overproduce uric acid.


Conjuntos de estudio relacionados

Factoring Trinomials: a > 1 / Assignment

View Set

Chapter 48: Diabetes Mellitus Lewis: Medical-Surgical Nursing, 11th Edition

View Set

Identify rational and irrational numbers

View Set

Chapter 5 T/F Review - Computer Programming

View Set

Series 66: Uniform Securities Act (Securities Registration)

View Set